Sei sulla pagina 1di 130

GIT-System

Wise 1700-by Sush and Team. 2016


Susmita, Asad, Manu, Saima, Zohaib, Savia, Shanu, Mona, Manisha, Sitara, Samreena, Sami and Komal




Dear Plabber,



• This first ever System Wise 1700 document was created thanks to 3 months of daily

hard work by the PLAB Skype group ‘Unity’ which was brought together by Dr Susmita

Chowdhury.


The team members were:


& Susmita (Lead/most ignorant as she is working full time in public health for 13 years)

& Asad (Invaluable in IT and all types of support/the heart of the group)

& Manu (Volunteered to solve more questions/pathologist/amazing genuine person)

& Saima (Most concise clear notes/ photographic memory)

& Zohaib (Great research/a surgeon)

& Savia (Great research/multi-tasker with two little ones)

& Shanu (Very helpful after her March exam for those appearing in June)

& Mona (Great contributor in discussions)

& Manisha (Gyne/great discussion contributor)

& Sitara (Good discussion contributor)

& Samreena (Stayed a shorter time but great)

& Sami (Contributed the most early on but too brilliant for the group/still great friends)

& Komal (Knowledgeable sweet supportive girl)


• The main purpose was to break down the 1700 Q Bank System wise.


• We did our own reliable research for the options (OHCM/Patient info etc.) and

concluded these keys below on skype. This can save you 100s of hours of research. But I

suggest you also do your own.


• 90% of the document consists of Unity research. We also added information from other

circulating documents and they are referenced as Dr Khalid/Dr Rabia (and her Team).


• However, several keys may be ‘incorrect’ and so please use your own judgment as we

take no responsibility. I suggest cross checking with Dr Khalid’s latest keys (a few of

which are still debatable). Finally decide on your own key.


• Sorry if some members failed to make their answers thorough. The highlights are mostly

as per what the team members wanted to highlight. Blank tables to be ignored.


• Note that some 1700 Questions are missing from here (when members did not do their

share). Questions may not be in order due to merging of documents and there is excess

information than required. Read as much as needed.


• This has been circulated by our team as a generous contribution to the Plabbers’ success

and must not be ‘sold’.


Good luck and best wishes: Sush and Team

1
GIT-System Wise 1700-by Sush and Team. 2016
Susmita, Asad, Manu, Saima, Zohaib, Savia, Shanu, Mona, Manisha, Sitara, Samreena, Sami and Komal




Q: 79 A woman who returned from abroad after 3 weeks of holiday complains of
severe diarrhea of 3 weeks. She also developed IDA Iron Deficiency Anaemia
and folic acid def. What condition best describes her situation?
a. Jejunal villous atrophy
b. Chronic diarrhea secretions
c. Malabsorption
d. Increased catabolism
e. Increased secretions of acid


Clincher(s) Diarrhoea, IDA and folic acid deficiency, abroad
A
B
C
D
E
KEY The key is C. Malabsorption.
Additional Malabsorbtion:
Information Signs of deficiency
• anaemia.(Fe, Folate deficiency or vitamin B12 deficiency.)
• Bleeding, (low vitamin K).
• Oedema, which occurs in protein/calorie malnutrition.
• Metabolic bone disease (<vit D)
• Neurological features (e.g. neuropathy)

R: (Tropical malabsorbtion) Tropical sprue is seen in residents of, and visitors

to, tropical areas (Middle East, Far East and the Caribeans) and it tends to

begin with an acute episode of diarrhoea, fever and malaise before settling

into a more chronic presentation of steatorrhea, malabsorption, nutritional

deficiency, anorexia, malaise and weight loss. Folate deficiency is a significant

part of the clinical picture.

Countries: Far and middle East, Caribbean

Rx: Tetracyclin and folic acid, optimum nutrition

2
GIT-System Wise 1700-by Sush and Team. 2016
Susmita, Asad, Manu, Saima, Zohaib, Savia, Shanu, Mona, Manisha, Sitara, Samreena, Sami and Komal



Reference OHCM 280 (Also read Coeliac disease, chronic pancreatitis)


Q: 88 A 45yo man with posterior gastric ulcer presented with severe excruciating
pain which subsided after conservative treatment. 10 days later he developed
swinging pyrexia. US shows a collection in the peritoneum. What will be the
most likely location of the collection?
a. Hepatorenal puch
b. Left paracolic gutter
c. Subphrenic
d. Pelvic cavity
e. Lesser sac


Clincher(s) Posterior gastric ulcer, severe pain, swinging pyrexia, collection in peritoneum.
(Complication of posterior gastric ulcer)
A
B
C
D
E
KEY Ans. The key is E. Lesser sac.

Additional
Information

The lesser sac,


also known as the omental bursa, is the cavity in the abdomen that is formed
by the lesser and greater omentum. Usually found in mammals, it is connected
with the greater sac via the omental foramen

3
GIT-System Wise 1700-by Sush and Team. 2016
Susmita, Asad, Manu, Saima, Zohaib, Savia, Shanu, Mona, Manisha, Sitara, Samreena, Sami and Komal


Swinging pyrexia: Due to Intra-abdominal sepsis: this is a term is used for any
intra-abdominal infection and encompasses both localised and
generalised peritonitis.

Perforation occurs when the ulcer erodes through the full thickness of the
gut wall. Gastric and duodenal contents spill into the peritoneal cavity
causing generalised peritonitis. The most frequent site of perforation is the
anterior wall of the first part of the duodenum.

Location: The clinical presentation of posterior perforations of gastric ulcers
depends on the location of these ulcers within the stomach. Ulcers situated in
the fundus or body of the stomach perforate into the lesser sac, with
consequent lesser sac abscess and generalized peritonitis with contamination
of the peritoneal cavity through the foramen of Winslow. (Ulcers in the
pylorus of the stomach perforate into the retroperitoneal space.) Posterior
perforations of gastric ulcers have a high mortality.

Symptoms: Classically, when a patient's peptic ulcer perforates, it floods his
peritoneum with the acid contents of his stomach, and gives him a sudden
agonizing pain. He may be able to tell you the moment the pain began; it is
constant, it spreads across his entire upper abdomen and later all over, and is
made worse by deep breathing or movement. Signs of peritonitis.
Approximately 10% of patients have no preceding history of dyspepsia. The
physical signs in the abdomen are dramatic. There is generalised tenderness,
guarding and rebound tenderness. The abdominal muscles are held rigid,
giving the classical ‘board-like rigidity’. Abdominal respiratory movements
and bowel sounds are absent. The percussion note over the liver may be
resonant because of free intraperitoneal air. In patients in whom the
perforations are sealed off by adjacent organs the signs may be localised to
the epigastrium. In other cases the spillage from the perforation may track
down the right paracolic gutter, resulting in maximal tenderness in the right
iliac fossa. This is the so-called ‘right paracolic gutter syndrome’ and may be
mistaken for acute appendicitis.

If he has perforated, he needs an urgent laparotomy.

Reference Need reference. Above from internet and research papers /see OHCM 608,
242


Q: 107 107. A 5m baby present with recurrent vomiting. Mother noticed some of the
vomitus is blood stained. Choose the single most likely inv?
a. Upper GI endoscopy
b. Barium meal
c. US

4
GIT-System Wise 1700-by Sush and Team. 2016
Susmita, Asad, Manu, Saima, Zohaib, Savia, Shanu, Mona, Manisha, Sitara, Samreena, Sami and Komal


d. Colonoscopy
e. CT abdomen


Clincher(s) Baby, recurrent vomiting/haematemesis
A
B
C
D
E
KEY The key is A. upper GI endoscopy.

Additional Endoscopy is the primary diagnostic investigation in patients with upper GI
Information bleeding.
Haematemesis (unless swallowed blood - eg, following a nosebleed or ingested
blood from a cracked nipple in some breast-fed infants) may suggest an
important and potentially serious bleed from the oesophagus, stomach or
upper gut.

Projectile vomiting, non bilious: Pyloric stenosis: Olive shape mass in
epigastrium
Bilious vomiting: Call for senior help, consider duodenal obstruction.

Because there is recurrent vomiting hematemesis, I think this is not childhood
Gastro-oesophageal reflux (GOR) which is the non-forceful regurgitation of
milk and other gastric contents into the oesophagus.


Reference Patient.info/ upper GI bleeding OHCM 252, 254, 830


Q: 110 A 35yo man presents with hx of dyspepsia. H.Pylori antibodies are negative. No
improvement is seen after 1m of tx. What is the next step?
a. Urea breath test
b. Gastroscopy
c. CT
d. MRI


Clincher(s) <50yrs, Long standing dyspepsia (Alarms sign), no H.Pylori,
A
B
C
D
E

5
GIT-System Wise 1700-by Sush and Team. 2016
Susmita, Asad, Manu, Saima, Zohaib, Savia, Shanu, Mona, Manisha, Sitara, Samreena, Sami and Komal


KEY Ans. 1. Gastroscopy. {to clarify: endo or gastro/ OHCM mentions endo)
Additional
Information Symptoms of dyspepsia: Alarm symptoms: Anaemia (iron defi ciency); loss of
weight; anorexia; recent onset/progressive symptoms;
melaena/haematemesis; swallowing difficulty.



Not responding to treatment D/D is: i) Zollinger Elison syndrome ii) Ca stomach

The following is description of the ALARMS signs and chart above:

Indications of gastroscopy in a 35 yo man (man of age <50yrs): i) Acute
symptoms with H/O previous episode (PUD) ii) Alarm features [weight loss,
anaemia, vomiting, hematemesis and melaena, dysphagia, palpable abdominal
mass], fear of cancer, evidence of organic disease.

6
GIT-System Wise 1700-by Sush and Team. 2016
Susmita, Asad, Manu, Saima, Zohaib, Savia, Shanu, Mona, Manisha, Sitara, Samreena, Sami and Komal


Urgent specialist referral - two-week rule


If the patient has dyspepsia at any age with any of the following alarm
symptoms:[13]
• Chronic GI bleeding.
• Progressive unintentional weight loss.
• Progressive dysphagia.
• Persistent vomiting.
• Iron-deficiency anaemia.
• Epigastric mass.
• Suspicious barium meal.
NB: patients aged 55 years or older with unexplained and persistent recent-
onset dyspepsia should be referred urgently for endoscopy

If age less than 55 and no alarm signs, try life style modifications and simple
antacids. If no improvement then do H.pylori testing (antibodies). If it is
positive do eradication and review in 4 weeks.
If resolved, no further action required.
If symptoms are not resolved, do urea breath test.
If it is positive, again eradication for H.pylori
If it is negative, do upper GI endoscopy

If the initial H.pylori testing was negative give PPIs or H2 blockers for 4 weeks
and review if symptoms resolve no action needed if they dont resolve do
upper GI endoscopy.

Reference OHCM 242


Q: 130 130. A 22yo woman with longstanding constipation has severe ano-rectal pain
on defecation. Rectal exam: impossible due to pain and spasm. What is the
most probable dx?
a. Anal hematoma
b. Anal fissure
c. Anal abscess
d. Protalgia fugax
e. Hemorrhoids


Clincher(s) Long standing constipation, severe an rectal pain on defecation, rectal exam
impossible
A
B
C

7
GIT-System Wise 1700-by Sush and Team. 2016
Susmita, Asad, Manu, Saima, Zohaib, Savia, Shanu, Mona, Manisha, Sitara, Samreena, Sami and Komal


D
E
KEY Ans. The key is B. Anal fissure.
Additional
Information Anal fissures: Acute If less than 6weeks, >6wks chronic.

Causes: Most are due to hard faeces. Spasm may constrict the inferior rectal
artery, causing ischaemia, making healing difficult and perpetuating the
problem.

History of constipation almost always present. Examination is almost


impossible due to severe pain.

Treatment: Acute: 1st line: Increase fluid intake, fiber diet. Bulk forming
laxatives are first line. Topical anesthetics are used. Lactulose can be tried.

1st line: lidocaine oint + GTN ointment or tropical diltiazem

Chronic: Topical GTN is the first line and mainstay of treatment. If ineffective
for >8wks surgical referral for (2nd line) use of botulinum toxin injection and
tropical diltiazem.

If above fail: surgery: lateral partial internal sphincterotomy


Reference OHCM 632

Q:143 A 3yo child who looks wasted on examination has a hx of diarrhea on and off.
The mother
describes the stool as bulky, frothy and difficult to flush. What is the single inv
most likely to
lead to dx?


Clincher(s) Young child, looks wasted, history of diarhhea, bulky frothy stool difficult to
flush.
A a. Sweat chloride test
B b. Anti-endomysial antibodies
C c. LFT
D d. US abdomen
E e. TFT
KEY The key is B. Anti-endomysial antibody

Additional Coeliac Disease: Suspect this in all those with diarrhoea + weight loss or
Information anaemia (esp. if iron or B12). It is a T-cell-mediated autoimmune disease of the
small bowel in which prolamin

8
GIT-System Wise 1700-by Sush and Team. 2016
Susmita, Asad, Manu, Saima, Zohaib, Savia, Shanu, Mona, Manisha, Sitara, Samreena, Sami and Komal


(alcohol-soluble proteins in wheat, barley, rye ± oats) intolerance causes
villous atrophy
and malabsorption (including of bile acids)

When people with celiac disease eat gluten (a protein found in wheat, rye and
barley), their body mounts an immune response that attacks the small
intestine. These attacks lead to damage on the villi, small fingerlike projections
that line the small intestine, that promote nutrient absorption. When the villi
get damaged, nutrients cannot be absorbed properly into the body.

Celiac disease is hereditary, meaning that it runs in families. People with a


first-degree relative with celiac disease (parent, child, sibling) have a 1 in 10
risk of developing celiac disease.


Investigations: FBC, Dec feritin, dec vit.B12
Antibodies: alpha -gliadin, transglutaminase and anti-endomysial 95% specific.
Duodenal biopsy shows subtotal villous atrophy.

9
GIT-System Wise 1700-by Sush and Team. 2016
Susmita, Asad, Manu, Saima, Zohaib, Savia, Shanu, Mona, Manisha, Sitara, Samreena, Sami and Komal



Long Term Health Effects

Celiac disease can develop at any age after people start eating foods or
medicines that contain gluten. Left untreated, celiac disease can lead to
additional serious health problems. These include the development of other
autoimmune disorders like Type I diabetes and multiple sclerosis (MS),
dermatitis herpetiformis (an itchy skin rash), anemia, osteoporosis, infertility
and miscarriage, neurological conditions like epilepsy and migraines, short
stature, and intestinal cancers.

Treatment
Currently, the only treatment for celiac disease is lifelong adherence to a strict
gluten-free diet. People living gluten-free must avoid foods with wheat, rye
and barley, such as bread and beer. Ingesting small amounts of gluten, like
crumbs from a cutting board or toaster, can trigger small intestine damage.

Read about the Gluten-Free Diet.

Celiac disease is also known as coeliac disease, celiac sprue, non-tropical sprue,
and gluten sensitive enteropathy.

Reference


Q: 144 A 45yo woman has had severe epigastric and right hypochondrial pain for a
few hours. She has a normal CBC, serum ALP is raised, normal transaminase. 3
months ago she had a
cholecystectomy done. What is the most appropriate inv?


Clincher(s) Epigastric and right hypochondrial pain. Alp raised, normal transaminase.
A a. US abdomen
B b. ERCP
C c. MRCP
D d. CT abdomen
E e. Upper GI endoscopy
KEY The key is B. ERCP. Diagnosis is choledocolithiasis
Additional Right upper quadrant pain… think of gall stones. And since the LFTs here show

10
GIT-System Wise 1700-by Sush and Team. 2016
Susmita, Asad, Manu, Saima, Zohaib, Savia, Shanu, Mona, Manisha, Sitara, Samreena, Sami and Komal


Information obstructive picture ALP increased with normal transaminases the obstruction
is most probably in the biliary tract CBD.
ERCP: Endoscopic retrograde cholangiopancreatography (ERCP)
Indications: No longer routinely used for diagnosis, it still has a significant
therapeutic role: sphincterotomy for common bile duct stones; stenting of
benign or malignant strictures and obtaining brushings to diagnose the nature
of a stricture.

MRCP: MRCP (magnetic resonance cholangiopancreatography) gives detail of
the biliary system and the pancreatic duct. MRCP has excellent sensitivity and
specificity for diagnosing common bile duct stones—when these are >6mm
both are 99% (although accuracy is lower for stones <6mm—and is the imaging
modality of choice.

But here since we need to remove the stones as well so we use ERCP.

Reference


Q: 156 A 12yo pt presents with copious diarrhea. Exam: urine output=low, mucous
membrane=dry, skin turgor=low. What is the most appropriate initial
management?


Clincher(s) Signs of dehydrtation.
A a. Antibiotic
B b. Antimotility
C c. Anti-emetic
D d. Fluid replacement
E e. Reassurance
KEY The key is D. Fluid replacement.
Diagnosis is severe dehydration.
Points in favour: i) low urine output ii) dry mucous membrane and iii) low skin
turgor.


Additional Gastroenteritis Rotavirus is the most common cause of gastroenteritis in
Information infants and children.
Norovirus (most common cause in adults)

ASSESSMENT OF DEHYDRATION:
Mild dehydration: Decreased urine output.
5% dehydration: Dry mucous membranes; decreased urine output.
10% dehydration: The above + sunken fontanele, inc pulse; hoarse cry; dec
skin turgor.

11
GIT-System Wise 1700-by Sush and Team. 2016
Susmita, Asad, Manu, Saima, Zohaib, Savia, Shanu, Mona, Manisha, Sitara, Samreena, Sami and Komal


>10%: The above, but worse, with: shock, drowsiness, and hypotension.

MANAGEMENT
Mild: Treated at home by oral rehydrating therapy.
Moderate: Oral fluids, via NG or IV fluids can be used. ‘Rapid rehydration’
involves 4 hours of
10mL/kg/h 0.9% NaCl then maintenance after if needed. Monitor U & Es
Severe: If not in shock oral or NG route can be used. If in shock. 0.9% saline
20mL/kg IVI bolus, while calculations are performed. Continuously monitor
pulse, BP, ECG. • Continue with boluses until the signs of shock ease.
• Then give the daily requirement + fluid deficit

Reference


Q: 216 A young woman complains of diarrhea, abdominal cramps and mouth ulcers.
AXR shows
distended transverse colon with goblet cell depletion on rectal biopsy. What is
the most
probable dx?


Clincher(s) Diarrhea, cramps, moutyh ulcers. Goblet cell depletion on rectal biopsy
A a. CD
B b. UC
C c. Bowel Ca
D d. Bowel obstruction
E e. IBS
KEY The key is B. UC.
In UC there is goblet cell depletion and less mucous production in contrast
with CD where there may be goblet cell hyperplasia and mucous secretion is
not reduced.

12
GIT-System Wise 1700-by Sush and Team. 2016
Susmita, Asad, Manu, Saima, Zohaib, Savia, Shanu, Mona, Manisha, Sitara, Samreena, Sami and Komal


Additional
Information


Reference


Q: 223 A 28 yo female presents with a 3m hx of diarrhea. She complains of abdominal
discomfort and passing stool 20x/day. Exam=febrile. Barium enema shows
cobblestone mucosa. What is the
most likely dx?


Clincher(s) Diarrhea, abdominal pain, febrile and cobblestone mucosa on barium enema.

13
GIT-System Wise 1700-by Sush and Team. 2016
Susmita, Asad, Manu, Saima, Zohaib, Savia, Shanu, Mona, Manisha, Sitara, Samreena, Sami and Komal


A a. Ameoba
B b. Colon Ca
C c. GE
D d. CD
E e. UC
KEY The key is D. CD. [Hx of diarrhea, abdominal discomfort, and patient being
febrile indicate gut inflammation and cobblestone appearance on barium
enema is suggestive of CD].

Additional
Information

14
GIT-System Wise 1700-by Sush and Team. 2016
Susmita, Asad, Manu, Saima, Zohaib, Savia, Shanu, Mona, Manisha, Sitara, Samreena, Sami and Komal


Reference

Q:374
A pt presents with longstanding gastric reflux, dysphagia and chest pain. On
barium enema, dilation of esophagus with tapering end is noted. He was found
with Barrett’s esophagus. He had progressive dysphagia to solids and then
liquids. What is the single most appropriate dx?
a. Achalasia
b. Esophageal spasm
c. GERD
d. Barrett’s esophagus
e. Esophageal carcinoma
Clincher(s) Barrett’s oesophagus, progressive dysphagia to solids then liquids, findings
on barium enema and long standing gastric reflux.
A Achalasia picture on barium enema can be similar but it's not associated with

15
GIT-System Wise 1700-by Sush and Team. 2016
Susmita, Asad, Manu, Saima, Zohaib, Savia, Shanu, Mona, Manisha, Sitara, Samreena, Sami and Komal


Barrett's oesophagus and dysphagia occurs with both solids and liquids.
Achalasia although can cause oesophageal carcinoma.
B Oesophageal spasm symptoms are intermittent, not associated with Barrett's
and barium swallow will show cork screw pic.
C Gord may induce Barrett's oesophagus but symptoms are not specific of gord.
D
E All the symptoms fit perfectly for this particular condition.
KEY E
Additional Achalasia on c X-ray : fluid level in esophagus above heart level.
Information Adenocarcinoma of esophagus is more common in Caucasians and in
developed countries whereas squamous carcinoma is commoner in
developing countries.
Reference Patient.info


Q:380
A pt presented with hx of swelling in the region of the sub-mandibular region,
which became more prominent and painful on chewing. He also gave hx of
sour taste in the mouth, the area is tender on palpation. Choose the most
probable dx?
a. Chronic recurrent sialadenitis
b. Adenolymphoma
c. Mikuliczs disease
d. Adenoid cystic carcinoma
e. Sub-mandibular abscess
Clincher(s) Swelling prominent and prominent chewing are characteristics features.
A Often due to stones in salivary glands.
B Aka Warthin’s Tumor is the benign cystic Tumor of parotid glan
C Benign persistent swelling of lacrimal and salivary glands and often ass with
sjogrens syndrome. Usually unilateral unless if Sjögren's syndrome is present
then can be bilateral enlargement of glands. OHCM page 720
D It's a rare type of cancer that exist in many different body sites including the
salivary glands. Here the particular symptoms related to Tumor are absent
E Symptoms of abscess would be increasing pain and pyrexia not confined only
when chewing and will will be persistent.
KEY A
Additional Plain sialography and treatment is removal of stones.
Information
Reference Ohcs page 578.


Q:389
A 60yo man presented with a lump in the left supraclavicular region. His
appetite is decreased and he has lost 5kg recently. What is the most probably
dx?

16
GIT-System Wise 1700-by Sush and Team. 2016
Susmita, Asad, Manu, Saima, Zohaib, Savia, Shanu, Mona, Manisha, Sitara, Samreena, Sami and Komal


a. Thyroid carcinoma
b. Stomach carcinoma
c. Bronchial carcinoma
d. Mesothelioma
e. Laryngeal carcinoma
Clincher(s) Anorexia, weight loss and left supraclavicular lymph node presence.
A Symptoms doesn't fit in apart from weight loss.
B Typical symptoms as described above.
C No symptoms ass with this cancer.
D Again the symptoms doesn't fit well with this tumor
E Same as above 2.
KEY B
Additional Common in blood group A. High salt or high nitrate, h pylori, pernicious
Information anemia, atrophic gastritis all ass with gastric carcinoma. Common in Asian
communities esp Japan and China. Deficiency of vit C and A also cause this
condition.
Virchow node that is large left supraclavicular lymph node is diagnostic of
gastric carcinoma. It can also spread to ovaries and specific term used for this
is “ krukenberg tumor”
Reference OHCM page 620


Q:392
. A 57yo man with blood group A complains of symptoms of vomiting,
tiredness, weight loss and palpitations. Exam: hepatomegaly, ascites, palpable
left supraclavicular mass. What is the most likely dx?
a. Gastric carcinoma
b. Colorectal carcinoma
c. Peptic ulcer disease
d. Atrophic gastritic
e. Krukenburg tumor

Clincher(s) Blood group A, palpitations due to anemia, vomiting,left supraclavicular
node
A Typical symptoms of gastric carcinoma
B Bkeeding /mucus PR, altered bowel habit etc are absent
C Peptic ulcer disease is not going to cause symptoms such as ascites,palpable
left supraclavicular node or hepatomegaly.
D Causes loss of gastric glands and secretions of hcl, pepsin and intrinsic factor
is impaired.
E Gastric carcinoma spread to ovaries.
KEY A
Additional Look description of qs 389
Information
Reference OHCM 620

17
GIT-System Wise 1700-by Sush and Team. 2016
Susmita, Asad, Manu, Saima, Zohaib, Savia, Shanu, Mona, Manisha, Sitara, Samreena, Sami and Komal




Q:394
A child presents with increasing jaundice and pale stools. Choose the most
appropriate test?
a. US abdomen
b. Sweat test
c. TFT
d. LFT
e. Endomyseal antibodie
Clincher(s) Increasing jaundice and pale stools.
A USG is
diagnostic in case of obstructive jaundice].
B It's for cystic fibrosis
C Can't exclude anything here.
D . [This is a picture suggestive of obstructive jaundice. LFT can give clue like
much raised bilirubin, AST and ALT not that high and raised alkaline
phosphatase.
E It's for Coeliac disease
KEY A
Additional Urobilinogen is absent in urine in obstructive jaundice. Main causes
Information cholangitis, pancreatic cancer, cholangiocarcinoma, stones.
Reference OHCM 250 and Dr khalid notes

Q: 331. An old lady 72yo staying at a nursing home for a few years, a known HTN on
reg tx presented with sudden dysphagia while eating with drooling of saliva
and req urgent inv. What would be
your next step?
a. Ba swallow
b. Chest CT
c. Endoscopy
d. Laryngoscopy
e. CXR
f. Endoscopy with biopsy

Clincher(s) New/Sudden Dysphagia, 72 old
A
B
C
D
E
KEY C
Additional 1.2 Upper gastrointestinal tract cancers
Information

18
GIT-System Wise 1700-by Sush and Team. 2016
Susmita, Asad, Manu, Saima, Zohaib, Savia, Shanu, Mona, Manisha, Sitara, Samreena, Sami and Komal


Oesophageal cancer

1.2.1 Offer urgent direct access upper gastrointestinal endoscopy (to be


performed within 2 weeks) to assess for oesophageal cancer in people:

• with dysphagia or
• aged 55 and over with weight loss and any of the following:
o upper abdominal pain
o reflux
o dyspepsia. [new 2015]

1.2.2 Consider non-urgent direct access upper gastrointestinal endoscopy to


assess for oesophageal cancer in people with haematemesis. [new 2015]

1.2.3 Consider non-urgent direct access upper gastrointestinal endoscopy to


assess for oesophageal cancer in people aged 55 or over with:

• treatment-resistant dyspepsia or
• upper abdominal pain with low haemoglobin levels or
• raised platelet count with any of the following:
o nausea
o vomiting
o weight loss
o reflux
o dyspepsia
o upper abdominal pain, or
• nausea or vomiting with any of the following:
o weight loss
o reflux
o dyspepsia
o upper abdominal pain. [new 2015]

Definition of Terms

Direct access When a test is performed and primary care retain clinical
responsibility throughout, including acting on the result.

Urgent To happen/be performed within 2 weeks.

Immediate An acute admission or referral occurring within a few hours, or


even more quickly if necessary.

Non-urgent The timescale generally used for a referral or investigation that is


not considered very urgent or urgent.

19
GIT-System Wise 1700-by Sush and Team. 2016
Susmita, Asad, Manu, Saima, Zohaib, Savia, Shanu, Mona, Manisha, Sitara, Samreena, Sami and Komal


o


Reference NICE: https://www.nice.org.uk/guidance/ng12/chapter/1-Recommendations-
organised-by-site-of-cancer#upper-gastrointestinal-tract-cancers
OHCM



Q: 362 . A 65yo known case of liver ca and metastasis presents with gastric reflux and

20
GIT-System Wise 1700-by Sush and Team. 2016
Susmita, Asad, Manu, Saima, Zohaib, Savia, Shanu, Mona, Manisha, Sitara, Samreena, Sami and Komal


bloatedness. On
bone exam there is osteoporosis. He also has basal consolidation in the left
lung. What is the
next appropriate step?
a. PPI IV
b. Alendronate
c. IV antibiotics
d. Analgesic
e. PPI PO

Clincher(s) Lung consolidation with gastric reflux. (Distractors: Liver ca and osteoporosis)
A
B is a bisphosphonate drug used for osteoporosis and several other bone
diseases.
C
D
E
KEY C Antibiotic because (dr. Khaled)
Additional Dr. Rabia: IN THIS case reflux is the cause of recurrent pneumonia so both C
Information AND E can be
right but to choose single one E is more appropriate
Reference Dr. Rabia




Q: 367 . A 32yo woman has severe right sided abdominal pain radiating into the groin
which
has lasted for 3h. She is writhing in pain. She has no abdominal signs. What is
the
most likely cause of her abdominal pain?
a. Appendicitis
b. Ruptured ectopic pregnancy
c. Salpingitis
d. Ureteric colic
e. Strangulated hernia

Clincher(s) “Loin to groin”: right side abdominal pain radiating into the groin.
Writhing in pain: ureteric colic one of the most painful medical conditions
No abdominal signs
A Right iliac fossa with rebound tenderness, there could be nausea, fever and
elevated wbc
B OHCS p262 Clinical presentation Always think of an ectopic in a sexually
active woman with abdominal pain; bleeding; fainting; or diarrhoea and
vomiting.

21
GIT-System Wise 1700-by Sush and Team. 2016
Susmita, Asad, Manu, Saima, Zohaib, Savia, Shanu, Mona, Manisha, Sitara, Samreena, Sami and Komal


Tubal colic causes abdominal pain which may precede vaginal bleeding. The
ectopic may rupture the tube with sudden severe pain, peritonism, and shock .
The patient may be faint, with a tender abdomen (95%), enlarged uterus
(30%), cervical excitation (50%), adnexal mass (63%). Presentation may just be
as diarrhoea and vomiting, or nausea and dizziness.
C OHCS 286 Salpingitis (infection affects the Fallopian tubes )Patients with acute
salpingitis may be most unwell, with pain, fever, spasm of lower abdominal
muscles (she may be most comfortable lying on her back with legs flexed) and
cervicitis with profuse, purulent, or bloody vaginal discharge. Heavy menstrual
loss suggests endometritis. Nausea and vomiting suggest peritonitis. Look for
suprapubic tenderness or peritonism, cervical excitation, and tenderness in the
fornices. It is usually bilateral, but may be worse on one side. Subacute
infection can easily be missed, and laparoscopy may be needed to make either
diagnosis
D
E Apparent hernia, tender, painful, red could be infected
KEY D
Additional Spiral non contrast CT is the Dx of choice for Renal stones
Information Dr. Rabia It indicate stone at lower ureter. [i) Pain from upper ureteral stones
tends to radiate to the flank and
lumbar areas. ii) Midureteral calculi cause pain that radiates anteriorly and
caudally. This midureteral pain in
particular can easily mimic appendicitis on the right or acute diverticulitis on
the left. iii) Distal ureteral stones
cause pain that tends to radiate into the groin or testicle in the male or labia
majora in the female.
Reference OHCM Renal stone P. 640, OHCS, Dr. Rabia

22
GIT-System Wise 1700-by Sush and Team. 2016
Susmita, Asad, Manu, Saima, Zohaib, Savia, Shanu, Mona, Manisha, Sitara, Samreena, Sami and Komal

23
GIT-System Wise 1700-by Sush and Team. 2016
Susmita, Asad, Manu, Saima, Zohaib, Savia, Shanu, Mona, Manisha, Sitara, Samreena, Sami and Komal


Q: 349. A 68yo male presented with swelling in the lower pole of the parotid
gland for the last 10yrs. Exam: firm in consistency. What’s the most probable
dx?
a. Pleomorphic adenoma
b. Adenolymphoma
c. Mikulicz’s disease
d. Parotiditis
e. Frey’s syndrome

Clincher(s) 68 y male, parotid swelling for 10 Y, firm
A Firm, middle age men, slow growing, treated by parotidectomy
B Old men, soft tumor, treated by enoculation
C WEBMED: NOT IMPORTANT Mikulicz syndrome is a chronic condition
characterized by the abnormal enlargement of glands in the head and neck,
including those near the ears (parotids) and those around the eyes (lacrimal)
and mouth (salivary). The tonsils and other glands in the soft tissue of the face
and neck may also be involved. Although the disorder is almost always
described as benign, it always occurs in association with another underlying
disorder such as tuberculosis, leukemia, syphilis, Hodgkin's disease,
lymphosarcoma, Sjögren syndrome, or lupus (SLE). People who have Mikulicz
syndrome are at heightened risk for developing lymphomas
D acute
E damage to or near the parotid glands redness and sweating on the cheek area
adjacent to the ear
KEY A

24
GIT-System Wise 1700-by Sush and Team. 2016
Susmita, Asad, Manu, Saima, Zohaib, Savia, Shanu, Mona, Manisha, Sitara, Samreena, Sami and Komal


Additional
Information


Reference OHCM 601


Q: 428. A 55yo man presents with hx of weight loss and tenesmus. He is dx with rectal
carcinoma.
Which risk factors help to develop rectal carcinoma except following?
a. Smoking
b. Family hx
c. Polyp
d. Prv carcinoma
e. High fat diet
f. High fibre diet

Clincher(s) Fibers are always good ☺ eat more salads
A
B

25
GIT-System Wise 1700-by Sush and Team. 2016
Susmita, Asad, Manu, Saima, Zohaib, Savia, Shanu, Mona, Manisha, Sitara, Samreena, Sami and Komal


C
D
E
KEY f
Additional
Information


Reference OHCM P 618

442, 465, Manu GIT
516, 540
442 A 55yo woman complains of retrosternal chest pain and dysphagia which
is intermittent and unpredictable. The food suddenly sticks in the middle
of the chest, but she can clear it with a drink of water and then finish the
meal without any further problem. A barium meal shows a ‘corkscrew
esophagus’. What is the single most likely dysphagia?
Clincher(s) Retrosternal chest pain, dysphagia, corkscrew esophagus
A Esophageal candidiasis- odynophagia/There may be a history of HIV or
other risk factors such as steroid inhaler use
B Esophageal carcinoma-Barium rat tail appearance, progressive dysphagia,
starts from solids
C Esophageal spasm- dysphagia, chest pain, regurgitation, corkscrew
esophagus
D Pharyngeal pouch- Contrast pool on barium swallow, no chest pain
E Plummer-vinson syndrome- iron deficiency, esophageal webs, burning
sensation in tongue

Additional KEY- C.
Information **Esophageal spasm- Oesophageal motility disorder. Dysphagia,
regurgitation and chest pain. corkscrew oesophagus on Barium swallow X-
ray. Ttt- Nitroglycerin, CCB, PPI. Botulinum toxin, balloon dilatation.

26
GIT-System Wise 1700-by Sush and Team. 2016
Susmita, Asad, Manu, Saima, Zohaib, Savia, Shanu, Mona, Manisha, Sitara, Samreena, Sami and Komal


**Plummer vinson syndrome- triad of iron deficiency, esophegeal webs
and dysphagia. Premalignant - squamous cell carcinoma of oesophegus.
Also presents with cheilitis, koilonychia, glossitis and splenomegaly.
Patient complains of burning sensation in tongue and oral mucosa. Ttt is
iron supplementation and endoscopic dilation for webs
**Oesophageal candidiasis- Immunocompromised like HIV or renal
transplant. Odynophagia, with oral thrush. maybe weight loss. Ttt
fluconazole for atleast 21 days or atleast 14 days after disappearance of
symptoms.
**Oesophageal carcinoma- Dysphagia to solid foods then later to liquids.
Weight loss, hoarseness of voice(if involving the recurrent laryngeal
nerve), hematemesis, hemoptysis, nausea and vomiting.
Risk factors- smoking and unhealthy diet.
Diagnosis- Endoscopy and biopsy.
Treatment- Surgery, radio and chemotherapy depending on stage.
**Pharyngeal pouch (Zencker’s diverticulum)- Common above 70. M:F is
5:1.
Presentation: Dysphagia, regurgitation, aspiration, chronic cough and
weight loss.Neck lump that gurgles on palpation. Halitosis from food
decaying in the pouch. Investigation: Barium swallow shows residual
contrast pool within the pouch. Aspiration from the pouch might cause
inhalation pneumonia. Ttt cricopharyngeal myotomy
Reference DR Rabia Notes
465 A 46yo man is being investigated for indigestion. Jejunal biopsy shows

27
GIT-System Wise 1700-by Sush and Team. 2016
Susmita, Asad, Manu, Saima, Zohaib, Savia, Shanu, Mona, Manisha, Sitara, Samreena, Sami and Komal


deposition of macrophages containing PAS (Periodic acid-schiff) +ve
granules. What is the most likely dx?
a. Bacterial overgrowth
b. Celiac disease
c. Tropical sprue
d. Whipple͛s disease
e. Small bowel lymphoma

Clincher(s) Pas positive granules
A Bacterial overgrowth

B Celiac disease- villous atrophy

C Tropical sprue – intraepithelial lymphocytes (autoimmune)

D Whipple’s disease- Per-iodic acid-schiff positive granules - pink granules
inside microphages

E Small bowel lymphoma- has lymphotocytes

Additional Periodic acid-schiff positive granules containing macrophages in jejunal
Information biopsy is diagnostic of whipple’s disease.
Coeliac disease is gluten sensitivity. Its an autoimmune disease.
Reference Dr Rabia Notes
516 A child complains of RIF pain and diarrhea. On colonoscopy, granular
transmural ulcers are seen near the ileo-cecal junction. What should be
the management?
Clincher(s) RIF Pain, diarrhea, with transmural ulcers- ileo cecal junction IBD
A Sulfasalazine
B Paracetamol
C Ibuprofen
D Metronidazole
E

Additional a. Sulfasalazine
Information Crohns disease-transmural ulcers
metronidazole is often used for isolated peri anal disease
Remission
glucocorticoids (oral, topical or intravenous) are generally used to induce
remission.
5-ASA drugs (e.g. mesalazine) are used second-line to glucocorticoids but
are not as effective
stopping smoking is a priority (remember: smoking makes Crohn's worse,
but may help ulcerative colitis)

28
GIT-System Wise 1700-by Sush and Team. 2016
Susmita, Asad, Manu, Saima, Zohaib, Savia, Shanu, Mona, Manisha, Sitara, Samreena, Sami and Komal


Maintenance
azathioprine or mercaptopurine is used first-line to maintain remission
methotrexate is used second-line

Reference
540 A 30yo lady complains of intermittent diarrhea, chronic abdominal and
pelvic pain and tenesmus. Sometimes she notices blood in her stool.
Select the most likely cause leading to her
symptoms?
Clincher(s) intermittent diarrhea, chronic abdominal and pelvic pain and tenesmus
with blood in stools
A Inflammatory bowel disease
B Diverticulosis
C Irritable bowel disease
D Adenomyosis
E UTI

Additional Apart from tenesmus all the other symptoms will occur in all diseases
Information • Diverticulosis is defined as the presence of diverticula which are
asymptomatic.
• The diagnosis of IBS should be considered if the patient has had
the following for at least 6 months:
• abdominal pain, and/or
• bloating, and/or
• change in bowel habit
Adenomyosis is the invasion of the myometrium by endometrial tissue.
Extrauterine endometrial tissue causes inflammation, pain and the
formation of adhesions. Clinically its significance is as a cause of chronic
pelvic pain, dyspareunia and female infertility.
Reference DR Rabia notes
Irritable bowel syndrome (IBS)IBD is not the same as irritable bowel
syndrome (IBS), which is a common condition that causes symptoms such
as:

• constipation
• diarrhoea
• abdominal pain
• http://www.nhs.uk/conditions/inflammatory-bowel-
disease/pages/introduction.aspx


Q:606 . A 65yo woman had an excision of colonic tumor 3yrs ago. Now she is losing weight an
feels
lethargic. Exam: pale but no abdominal findings. What is the most appropriate inv?
a. CA 125

29
GIT-System Wise 1700-by Sush and Team. 2016
Susmita, Asad, Manu, Saima, Zohaib, Savia, Shanu, Mona, Manisha, Sitara, Samreena, Sami and Komal


b. CA 153
c. CA 199
d. CEA
e. AFP



Clincher(s) Age, history of colonic CA, weight loss

A CA 125 = ovarian cancer
B CA 153 = cancer breast
C CA 199 = pancreatic cancer
D CEA = colorectal carcinoma,
E AFP = hepatocellular carcinoma
KEY D
Additional This is the 3rd most common cancer and 2nd most common cause of UK cancer deaths
information
(16,000 deaths/yr). Usually adenocarcinoma. 86% of presentations are in those

>60yrs old















Frequently used tumour markers50

30
GIT-System Wise 1700-by Sush and Team. 2016
Susmita, Asad, Manu, Saima, Zohaib, Savia, Shanu, Mona, Manisha, Sitara, Samreena, Sami and Komal


Tumour Relevant Use Associated cancers Associated benign conditions
marker cancer
Other cancers and benign conditions in which the same tumour
marker may be raised (not comprehensive)
Alpha- Germ Diagnosis, Colorectal; gastric; Cirrhosis; pregnancy; neural tube
fetoprotein cell/testicular prognosis, moni- hepatobillary; lung defects
(FP)
Hepatocellular toring treatment
& detecting
recurrence
Calcitonin Medullary Diagnosis, None known C-cell hyperplasia
thyroid monitoring treat-
ment & detecting
recurrence
Cancer antigen Ovarian Diagnosis, Breast; cervical; endometrial; Many: Liver disease; cystic fibrosis;
(CA) prognosis, moni- hepatocellular; lung; non- pancreatitis; uri- nary retention;
125 toring treatment Hodgkin’s lymphoma; diabetes; heart failure; pregnancy;
& detecting pancreatic; medullary SLE; sarcoid; RA; diverticulitis; IBS;
recurrence thyroid carcinoma; endometriosis; fibroids
peritoneal; uterine
CA 19-9 Pancreatic Diagnosis, Colorectal; gastric; Acute cholangitis; cholestasis;
prognosis, moni- hepatocellular; pancreatitis; diabetes;
toring treatment oesophageal; ovarian IBS; jaundice
& detecting
recurrence
CA 15-3 Breast Monitoring Hepatocellular; pancreatic Cirrhosis; benign breast disease; in
treatment & normal health
detecting
recurrence
Carcinoemb Colorectal Prognosis, Breast; gastric; lung; Smoking; chronic liver disease; chronic
ryonic monitoring treat- mesothelioma; kidney disease;
antigen ment & detecting oesophageal; pancreatic diverticulitis; jaundice
(CEA) recurrence
Human Germ Diagnosis, Lung Pregnancy
chorionic gon-
adotrophin ( - cell/testicular prognosis, moni-
HCG) toring treatment
Gestational
troph- & detecting
oblastic recurrence
Paraproteins B cell Diagnosis, None known None known
proliferative monitoring treat-
disor- ders (eg ment & detecting
myeloma) recurrence
Thyroglobulin Thyroid Monitoring None known None known
(follicular/papill treatment &
ary) detecting

31
GIT-System Wise 1700-by Sush and Team. 2016
Susmita, Asad, Manu, Saima, Zohaib, Savia, Shanu, Mona, Manisha, Sitara, Samreena, Sami and Komal


recurrence




Oncology and palliative care


Reference OHCM 9th edition,page 535


• Q:625 625. A 25yo woman is presenting with diarrhea and abdominal bloating over
the last 4m. Exam: she has blistering rash over her elbows. Biochemistry: low
serum albumin, calcium and folate conc.
On jejunal biopsy, there is shortening of the villi and lymphocytosis. What is
the most likely dx?
a. Celiac disease
b. Whipple’s disease
c. Crohn’s disease
d. Tropical sprue
e. Giardiasis
f. Cystic fibrosis

Clincher(s) diarhea,bloating,blistering rash,shortening of villi
A.Celiac Disease Suspect this in all those with diarrhoea + weight loss or anaemia (esp. if iron or
B12.
It is a T-cell-mediated autoimmune disease of the small bowel in which
prolamin(alcohol-soluble proteins in wheat, barley, rye ± oats) intolerance
causes villous atrophy and malabsorption .
Association: autoimmune disease; dermatitis herpetiformis,HLADQ2(95%)
Typical age: Any (peaks in infancy and 50–60yrs)
There is a 10% prevalence in 1st-degree relatives and a 30% relative risk for
siblings.
Presentation Stinking stools/steatorrhoea; diarrhoea; abdominal pain;
bloating;nausea + vomiting; aphthous ulcers; angular stomatitis weight
loss,fatigue;
weakness; osteomalacia; failure to thrive (children).
Diagnosis: Antibodies: Alpha gliadin, transglutaminase
and anti-endomysial—an IgA anti body, 95% specific unless patient is IgA-defi
cient. Duodenal biopsy done at endoscopy(as good as jejunal biopsy). Subtotal
villous atrophy, increased intra-epithelial WBCS + crypt hyperplasia, reversing
on gluten-free diet.
Treatment Lifelong gluten-free diet (ie no prolamins)
B.Whipples disease 1.Usually observed in middle age white men (more than 40 yrs).
2.Systemic disease caused by gram positive bacteria, Tropheryma Whippeli.
3.Clinical manifestions caused by infiltration of various body tissues by

32
GIT-System Wise 1700-by Sush and Team. 2016
Susmita, Asad, Manu, Saima, Zohaib, Savia, Shanu, Mona, Manisha, Sitara, Samreena, Sami and Komal


T.whippeli.
4.Classic presentation is signs of wasting illness(arthralgias,arthritis,fever
diarhea and other systemic symptoms depending on the organs involved like
GI bleeding etc.


E
KEY A
Additional Complications
Information • anaemia: iron, folate and vitamin B12 deficiency (folate deficiency is
more common than vitamin B12 deficiency in coeliac disease)
• hyposplenism
• osteoporosis, osteomalacia
• lactose intolerance
• enteropathy-associated T-cell lymphoma of small intestine
• subfertility, unfavourable pregnancy outcomes

Reference OHCM,emedicine.medscape


Q:633 633. A 56yo woman has had severe abdominal pain for 24h radiating to her
back and is accompanied by nausea and vomiting. She appears to be
tachycardic and in shock. She was found to have gallstones, 2yrs ago. What
is the most likely inv to confirm dx?
a. US abdomen
b. LFT
c. Serum lipase
d. Angiography
e. CT abdomen

Clincher(s) Acute Severe pain radiating to back,patient in shock,history of gall stones
A
B
C
D
E
KEY C
Additional Acute Pancreatitis:pancreatic inflammation by enzyme-mediated
Information autodigestion; Oedema and fluid shifts causing hypovolaemia, as
Acute Pancreatitis extracellular fl uid is trapped in the gut, peritoneum, and retroperitoneum
(worsened by vomiting). Progression may be rapid from mild oedema to
necrotizing pancreatitis. ~50% of cases that advance to necrosis are further
complicated by infection. Pancreatitisis mild in 80% of cases; 20% develop
severe complicated and life-threatening disease.
Reference

33
GIT-System Wise 1700-by Sush and Team. 2016
Susmita, Asad, Manu, Saima, Zohaib, Savia, Shanu, Mona, Manisha, Sitara, Samreena, Sami and Komal




Q:648 648. A 6wk child with profuse projectile vomiting. What is the first thing you
will do?
a. US
b. Check serum K+ level
c. ABG
d. NG tube
e. IV fluids


Clincher(s) Profuse projectile vomiting
A
B
C
D
E
KEY B
Additional Presents at 3-8 weeks with vomiting which occurs after feeds, and becomes
Information projectile (eg vomiting over far end of cot). Pyloric stenosis is distinguished
from other causes of
vomiting by the following:

• The vomit does not contain bile, as the obstruction is so high.

• No diarrhoea: constipation is likely (occasionally ‘starvation stools’).

• Even though the patient is ill, he is rarely obtunded: he is alert, anxious,
and always hungry—and possibly malnourished, dehydrated.

• The vomiting is extremely large volume and within minutes of a feed.
Imaging:

Ultrasound detects early, hard-to-feel pyloric tumours, but is only needed if
examination is –ve.
Barium studies are ‘never’ needed.
Management:
Correct electrolyte disturbances. Before surgery (Ramstedt’s pyloromyotomy/
endoscopic surgery)pass a wide bore nasogastric tube.



Reference OHCM

34
GIT-System Wise 1700-by Sush and Team. 2016
Susmita, Asad, Manu, Saima, Zohaib, Savia, Shanu, Mona, Manisha, Sitara, Samreena, Sami and Komal


Q:671 671. A 4m girl has severe FTT failure to thrive and increasing jaundice which
was 1st noticed at 1wk of age. She has an enlarged liver and scratches on her
skin. Her parents have been unable to seek medical care.
What is the most likely dx?
a. Biliary atresia
b. G6PD deficiency
c. Hep B
d. Spherocytosis



Clincher(s) Increasing jaundice,scratches
A


B G6PD def is hereditary disease of XLR inheritance and always presented when
aggravating factor present (some medication) and otherwise asymptomatic
C
D congenital spherocytosis not presented with this early stage upto
development of gallstone due to haemolysis.

E
KEY A
Additional .
Information
Reference OHCM,Dr,Rabia notes

Q: 430 A 45yo woman complains of pain in her hands precipitated by exposure to the
cold weather. She is breathlessness on walking. When she is eating, she can

35
GIT-System Wise 1700-by Sush and Team. 2016
Susmita, Asad, Manu, Saima, Zohaib, Savia, Shanu, Mona, Manisha, Sitara, Samreena, Sami and Komal


feel food suddenly sticking to the gullet. It seems to be in the middle of the
esophagus but she can’t localize exactly where it sticks. It is usually relieved
with a drink of water. Choose the single most likely cause of dysphagia from
the options?
a. Esophageal carcinoma
b. Systemic sclerosis
c. SLE
d. Pharyngeal carcinoma
e. Globus hystericus

Clincher(s) pain in her hands due to cold weather, SOB, dysphagia relieved with drink of
water
A & D ruled out simply because the pt. presents with systemic complaints, and these
two will cause only local signs.
C SLE has raynauds phenomenon but no dysphagia; typically, in women of child
bearing age. OHCM pg: 556
E Globus hystericus is when a patient feels like they have a lump in their throat,
when infact they don’t. Examination is completely normal.
KEY : B Systemic sclerosis (SS) is thickening of the skin caused by accumulation of
collagen, and by injuries to the smallest arteries. It is classified into 2 types-
Limited cutaneous SS (70%) and Diffuse cutaneous SS (30%) according to
extent of skin involvement.
• Limited SS:
" Skin involvement limited to face, hands and feet
" CREST Syndrome:
∼ Calcinosis (subcutaneous tissues)
∼ Raynaud’s phenomenon- cardinal sign, early and very
common presentation.
∼ Esophageal dysmotility
∼ Sclerodactyly
∼ Telangiectasia
" Pulmonary hypertension: s/s >> SOB, fatigue, non- productive
cough, dizzy, chest pain, palpitations, peripheral oedema,
• Diffuse SS:
" Diffuse skin involvement (whole body in severe cases) and
organ fibrosis: lung, cardiac and renal dz.

Additional Management of SS: Currently no cure. Immunosuppressive regimens, including
Information IV cyclophosphamide, are used for organ involvement or progressive skin
disease. Trials of antifibrotic tyrosine kinase inhibitors are ongoing. Monitor BP
and renal function. Regular ACE-i or A2RBS risk of renal crisis (p314). Raynaud’s
phenomenon: (see p722) hand warmers, stop smoking, CCBs (nifedipine) or
intermittent IV prostacyclin
Reference OHCM pg: 554

36
GIT-System Wise 1700-by Sush and Team. 2016
Susmita, Asad, Manu, Saima, Zohaib, Savia, Shanu, Mona, Manisha, Sitara, Samreena, Sami and Komal




Q:540 A 30yo lady complains of intermittent diarrhea, chronic abdominal and pelvic
pain and tenesmus. Sometimes she notices blood in her stool. Select the most
likely cause leading to her symptoms?
a. Inflammatory bowel disease
b. Diverticulosis
c. Irritable bowel disease
d. Adenomyosis
e. UTI

Clincher(s) intermittent diarrhea, chronic abdominal and pelvic pain, tenesmus.
sometimes blood in her stool
B Diverticulosis is defined as the presence of diverticula which are
asymptomatic. When they have symptoms it is LLQP that is colicky in nature.
Diverticulitis – fever, tenderness.
C The diagnosis of IBS (irritable bowel syndrome) should be considered if the
patient has had the following for at least 6 months:
• abdominal pain (or discomfort) is either relieved by defecation and/or
• change in bowel habit
• and there are ≥2 of: urgency; incomplete evacuation; abdominal
bloating/distension; mucous per rectum (PR); worsening of symptoms after
food.
All examinations and investigations are normal therefore it is diagnosis by
exclusion (samsons notes)
D Adenomyosis is the invasion of the myometrium by endometrial tissue. Extra-
uterine endometrial tissue causes inflammation, pain and the formation of
adhesions. Clinically its significance is as a cause of chronic pelvic pain,
dyspareunia and female infertility – no diarrhea
E UTI – no diarrhea
KEY: A Inflammatory bowel disease
S/S: Episodic or chronic diarrhoea (± blood & mucus); crampy abdominal
dis comfort; bowel frequency relates to severity; urgency/tenesmus ≈ rectal
UC. Systemic symptoms in attacks: fever, malaise, anorexia, weight loss.
Extraintestinal signs: Clubbing; aphthous oral ulcers; erythema
nodosum; pyoderma gangrenosum; conjunctivitis; episcleritis; iritis; large
joint arthritis; sacroiliitis; ankylosing spondylitis; fatty liver; PSC;
cholangiocarcinoma; nutritional deficits; amyloidosis.

37
GIT-System Wise 1700-by Sush and Team. 2016
Susmita, Asad, Manu, Saima, Zohaib, Savia, Shanu, Mona, Manisha, Sitara, Samreena, Sami and Komal


Additional
Information


Investigation:
• Faecal calprotectin testing is recommended by NICE as an option to
help doctors distinguish between inflammatory bowel diseases, such as
Crohn’s disease and ulcerative colitis, and non-inflammatory bowel
diseases, such as irritable bowel syndrome
• If perforation do erect CXR
• Ba enema: Never do during severe attacks or for diagnosis
• Colonoscopy shows disease extent and allows biopsy - look for
inflammatory infiltrate; goblet cell depletion; glandular distortion;
mucosal ulcers; crypt abscesses.
Reference OHCM pg:272, NICE



Q: 551 A 40yo woman complains of dysphagia for both solids and liquids. She
sometimes suffers from severe retrosternal chest pain. Barium swallow reveals
a dilated esophagus which tapers to a fine distal end. What is the best
management strategy?
a. Reassurance
b. Antispasmodics
c. Dilatation of the LES
d. Endoscopic diverticulectomy
e. Barium swallow

Clincher(s) dysphagia for both solids and liquids, severe retrosternal chest pain, Ba
enema: dilated esophagus which tapers to a fine distal end
A Reassurance – asking for a management
B Antispasmodics – short term management
C
D Endoscopic diverticulectomy – for pharyngeal pouch
E Barrium swallow - investigation
KEY: C Dilation of the LES

Achalasia: The lower oesophageal sphincter fails to relax (due to degeneration
of the myenteric plexus), causing dysphagia (for fluids and solids),
regurgitation, substernal cramps, and ↓weight.

38
GIT-System Wise 1700-by Sush and Team. 2016
Susmita, Asad, Manu, Saima, Zohaib, Savia, Shanu, Mona, Manisha, Sitara, Samreena, Sami and Komal


• Achalasia typically presents in middle-age and is equally common in
men and women

Investigations
• manometry: excessive LOS tone which doesn't relax on swallowing -
considered most important diagnostic test
• barium swallow shows grossly expanded oesophagus, fluid level, 'bird's
beak' appearance. This is in contrast to the rat's tail appearance of
carcinoma of the oesophagus
• CXR: wide mediastinum, fluid level
• Gold standard - Manometery

Treatment: endoscopic balloon dilatation, or Heller’s cardiomyotomy—then
proton pump inhibitors (PPIS, p244). Botulinum toxin injection if a non-invasive
procedure is needed (repeat every few months). Calcium channel blockers and
nitrates also relax the sphincter.
• intra-sphincteric injection of botulinum toxin
• Heller cardiomyotomy for fit young patients.
• balloon dilation for old unwell patients.

Complications: Aspiration pneumonia, perforation, GERD, Oesophagus CA

Additional Dr. khalid>> The key is C. Dilatation of LES. [Dysphagea for both solids and
Information liquids suggest neuromuscular dysphagia while dysphagia only for solid
suggests mechanical obstruction. Here features are consistent with achalasia
for which lower oesophageal sphincter dilation (balloon dilatation) is a
treatment modality].
Reference Dr. rabia, OHCM pg: 240


Q: 564 A 45yo man with colon cancer now develops increased thirst, increased
frequency in urination and weight loss. His fasting blood glucose=9mmol/L.
what is the most appropriate management?
a. Oral hypoglycemic
b. Insulin long acting
c. Short acting insulin before meal
d. IV insulin
e. Subcutaneous insulin

Clincher(s)
A
B
C
D
E

39
GIT-System Wise 1700-by Sush and Team. 2016
Susmita, Asad, Manu, Saima, Zohaib, Savia, Shanu, Mona, Manisha, Sitara, Samreena, Sami and Komal


KEY: A Oral hypoglycemic
colon cancer is assoc with hyperinsulinemia or insulin resistance. so oral
hypoglycemics preferred because oral hypoglycemic (metformin) has
anticancerogenic effect.
A. first line treatment don’t get confused by colon cancer.

Metformin is the first drug of choice for the management of type 2 diabetes. It
has two main antidiabetic mechanisms of action, both of which have also been
implicated as anticarcinogenic mechanisms. Firstly, metformin inhibits hepatic
glucose production through an LKB1/AMP-activated protein kinase–mediated
mechanism which has been shown to adversely affect the survival of cancer
cell lines. Secondly, metformin improves insulin sensitivity in peripheral tissues
reducing hyperinsulinemia. Insulin resistance and hyperinsulinemia have been
associated with increased risk of several types of neoplasm and specifically
with colorectal cancer.


Normal fasting glucose: 3.5 – 5.5 mmol/L

Dr. Khalid >> . The key is A. Oral hypoglycemic. [Increased thirst and increased
frequency in urination along with weight loss is suggestive of DM supported by
fasting blood glucose of 9 mmol/L. At the age of 45 most likely type of diabetes
is NIDDM or type 2 DM which is treated by oral hypoglycemic agents].

Additional Colorectal carcinoma: 2nd most common cause of UK cancer deaths. Usually
Information adenocarcinoma. 86% of presentations are in those >60yrs old.
Presentation depends on site:
Left-sided: Bleeding/mucus PR; altered bowel habit or obstruction (25%);
tenesmus; mass PR (60%).
Right: Weight↓; Hb↓; abdominal pain; obstruction less likely.
Either: Abdominal mass; perforation; haemorrhage; fistula.
Reference Colorectal carcinoma - OHCM pg: 618, DM – OHCM pg: 200


Q: 576 A 40yo man with marked weight loss over the preceding 6m has bilateral
white, vertically corrugated lesion on the lateral surfaces of the tongue. What

40
GIT-System Wise 1700-by Sush and Team. 2016
Susmita, Asad, Manu, Saima, Zohaib, Savia, Shanu, Mona, Manisha, Sitara, Samreena, Sami and Komal


is the single most likely dx?
a. C1 esterase deficiency
b. Crohns disease
c. HIV disease
d. Sarcoidosis
e. Sjogren’s syndrome

Clincher(s)
A C1 esterase deficiency [hereditary angioedema (HA)]: autosomal dominant,
causes recurrent episodes of angioedema in the upper respiratory,
gastrointestinal tract or in subcutaneous tissues.
B CD – aphthous ulcers
C
D Sarcoidosis – erythema nodosum in acute cases, Usually affects adults aged
20–40yrs,more common in women. African–Caribbeans are affected more
frequently and more severely than Caucasians, particularly by extra-thoracic
disease
E Sjögren’s syndrome: ↓tear production (dry eyes, kerato-conjunctivitis sicca),
↓salivation (xerostomia—dry mouth, caries), parotid swelling. Other glands are
affected causing vaginal dryness,dyspareunia, dry cough and dysphagia
KEY: C HIV disease
'Hairy' leukoplakia is a shaggy white patch on the side of the tongue seen in
HIV, caused by EBV.

The natural history of hairy leukoplakia is variable. Lesions may frequently
appear and disappear spontaneously. Hairy leukoplakia is often asymptomatic
and many patients are unaware of its presence. Some patients with hairy
leukoplakia do experience symptoms including mild pain, dysaesthesia,
alteration of taste and the psychological impact of its unsightly cosmetic
appearance.
• Systemic antiviral therapy, which usually achieves resolution of the
lesion within 1-2 weeks of therapy.
• Topical therapy with podophyllin resin 25% solution, which usually
achieves resolution after 1-2 treatment applications.
• Topical therapy with retinoic acid (tretinoin), which has been reported
to resolve hairy leukoplakia.
• Ablative therapy, which can also be considered for small hairy
leukoplakia lesions. Cryotherapy has been reported as successful but is
not widely used.

Dr. Khalid>> The key is C. HIV disease. [The lesion described is leukoplakia
which is likely association of HIV disease].
Additional
Information
Reference HA -

41
GIT-System Wise 1700-by Sush and Team. 2016
Susmita, Asad, Manu, Saima, Zohaib, Savia, Shanu, Mona, Manisha, Sitara, Samreena, Sami and Komal


http://www.rch.org.au/clinicalguide/guideline_index/C1_Esterase_Inhibitor_D
eficiency/

Q: 580 . An 83yo woman who is a resident in a nursing home is admitted to hospital
with a 4d hx of
diarrhea. She has had no weight loss or change in appetite. She has been on
analgesics for 3wks
for her back pain. She is in obvious discomfort. On rectal exam: fecal
impaction. What is the
single most appropriate immediate management?
a. Codeine phosphate for pain relief
b. High fiber diet
c. Oral laxative
d. Phosphate enema
e. Urinary catheterization



Clincher(s) AGE , nursing home ,analgesic use
A Codeine phosphate – not as immediate management
B High fiber diet – increases gut motility
C Oral lax – increases gut motility , not used as immediate
D Enema – is the immediate rx to relieve the obs
E
KEY D- PHOSPHATE ENEMA

21

Additional Codeine phosphate SE: constipation, hence contraindicated in this patient.
Information (Codeine phosphate).
Bulk producers:
• Increase faecal mass, which stimulates peristalsis.
Contra-indications: difficulty in swallowing; intestinal obstruction faecal
impaction.
Stool softeners:
• Side-effects can include: anal seepage, lipoid pneumonia,
malabsorption of fat-soluble vitamins
Stimulants:
• Increase intestinal motility and should not be used in intestinal
obstruction.
• Prolonged use should be avoided, as it may cause colonic atony and
hypokalaemia (but there are no good, long-term follow-up studies).
Osmotic agents:

42
GIT-System Wise 1700-by Sush and Team. 2016
Susmita, Asad, Manu, Saima, Zohaib, Savia, Shanu, Mona, Manisha, Sitara, Samreena, Sami and Komal


• Retain fluid in the bowel.
Enemas and suppositories - useful additional treatment.

Reference Pg 248


Q:582 A 78yo woman presents with unilateral headache and pain on chewing. ESR=
70mm/hr. She is on oral steroids. What is the appropriate additional tx?
a. Bisphosphonates
b. HRT hormonal replacement therapy
c. ACEi
d. IFN
e. IV steroids


Clincher(s) Age and signs of temporal arteritis
A Bisphosphonate
B HRT
C ACE
D IFN
E IV STEROIDS
KEY . Bisphosphonates
She getting treated for temporal arteritis, therefore steroid will cause
osteoporosis. So additional therapy is A
We first do ESR and start steroids. If symptoms are not resolved then we up
the increase dose of steroids.
Additional treatment would be bisphosphonate to reduce risk of osteoporosis.



Additional
Information Bisphosphonates
Bisphosphonates decrease demineralisation in bone. They inhibit osteoclasts.

Clinical uses
• prevention and treatment of osteoporosis
• hypercalcaemia
• Paget's disease
• pain from bone metatases

Adverse effects
• oesophagitis, oesophageal ulcers
• osteonecrosis of the jaw
• increased risk of atypical stress fractures of the proximal femoral shaft.

43
GIT-System Wise 1700-by Sush and Team. 2016
Susmita, Asad, Manu, Saima, Zohaib, Savia, Shanu, Mona, Manisha, Sitara, Samreena, Sami and Komal



In longterm use of steroids prevent osteoporosis by ; exercise,
bisphosphonate,calcium ,vitd suppl,smoking cessation adv

Do not stop steroids abruptly when longterm use or does 7.5mg/day as


adrenal suff can occur .other indications in ochm pg 370

But if used `less than 3 weeks time – can be stopped immediately o


Reference Ohcm pg 370


Q:591 A 58yo pt presents with altered bowel habits and bleeding per rectum. Exam
and sigmoidoscopy showed an ulcer. What is the single most likely dx?
a. Colorectal carcinoma
b. Celiac disease
c. Crohn's disease
d. UC
e. IBS


Clincher(s) Age , alt bowel habits , bleeding PR , ulcer on sigmoidoscopy
A Colorectal ca – most likely
B Celiac disease
C Crohn disease
D Uc
E Ibs
KEY Colorectal carcinoma
Older patient. Altered bowel habits and bleeding PR is Ca unless proven
otherwise.
Alter bowl habbit+ bleeding are a red flag for CA colon>>>> colonoscopy >>>>
one ulcer >>> CA.in UC Usually there is no alteration in bowel habit, usually
bloody diarrhoea >>> colonoscopy >>>> multiple ulceration

Additional third most common cancer in the UK
Information second most common cause of cancer death in the UK.

Presentation:
Right colon cancers: weight loss, anaemia, occult bleeding, mass in right iliac
fossa, disease more likely to be advanced at presentation.
Left colon cancers: often colicky pain, rectal bleeding, bowel obstruction,
tenesmus, mass in left iliac fossa, early change in bowel habit, less advanced
disease at presentation.
The most common presenting symptoms and signs of cancer or large polyps
are rectal bleeding, persisting change in bowel habit and anaemia.

44
GIT-System Wise 1700-by Sush and Team. 2016
Susmita, Asad, Manu, Saima, Zohaib, Savia, Shanu, Mona, Manisha, Sitara, Samreena, Sami and Komal


Colonoscopy is the gold standard for diagnosis of colorectal cancer.


Reference


Q: 594 . A 72yo man presents with intermittent difficulty in swallowing with
regurgitation of stale food
materials. Sometimes he wakes up at night with a feeling of suffocation.
Choose the single most
likely cause of dysphagia?



Clincher(s) Age, intermittent dysphagia
A a. Benign structure
B b. Esophageal carcinoma

C Esophageal spasm
D pharyngeal pouch
E Systemic sclerosis
KEY d. Pharyngeal pouch

Pharyngeal More common in older men
pouch Represents a posteromedial herniation between thyropharyngeus an
cricopharyngeus muscles

Additional Usually not seen but if large then a midline lump in the neck that gurgles on
Information palpation
Typical symptoms are dysphagia, regurgitation, aspiration and chronic cough.
Halitosis( bad breath ) may occasionally be seen
Reference


Q:597 597. A 36yo pt came with diarrhea, bleeding, weight loss and fistula. What is
the single most likely dx?
a. Celiac disease
b. Crohns disease
c. UC
d. IBS

.

45
GIT-System Wise 1700-by Sush and Team. 2016
Susmita, Asad, Manu, Saima, Zohaib, Savia, Shanu, Mona, Manisha, Sitara, Samreena, Sami and Komal




Clincher(s) Diarrhea , bleeding , wt loss , fistula
A Celiac -
B Crohns-
C UC
D IBS
E
KEY b. Crohns disease
fistula due to transmural involvement, weight loss in crohns
Additional
Information
Reference

Q: 731 An old man having T2DM with increased skin tanning, heart
failure and having high ferritin (hemochromatosis) level is refusing
tx. Where is the first site of cancer?


a. Testes


b. Adrenal


c. Liver


d. Pancreas


Clincher(s) h/o increased tannin, high ferritin level.
A Testicular failure occur in Heamochromatosis due to the pituitary
hypo functioning without any direct involvement of testis.
B Hyporeninaemic hypoaldosteronism is endochrinopathies of
hemochromatosis in which there is a decrease production of
angiotensin 2 along with intraadrenal dysfucntion.

46
GIT-System Wise 1700-by Sush and Team. 2016
Susmita, Asad, Manu, Saima, Zohaib, Savia, Shanu, Mona, Manisha, Sitara, Samreena, Sami and Komal


C Hepatomegaly, cirrhosis and chronic liver disease occur as
manifestation of hemochromosis. If cirrhosis is present, the
chances of HCC(hepatocellular carcinoma carcinoma) is 22-33%
especially if the patient is age above 50years, HBsAg positive or
alcholic. The heart failure in this patient is due to the dilated
cardiomyopthay(decreased left ventricular function due to
enlarged cavity and decrease pumping action).

D Bronze Diabetes mellitus occurs due to the deposition of iron in


pancreas.

KEY C- liver.
Additional • Hereditary disorder of iron metabolism in which increase
Information(accordin intestinal absorption of iron leads to the deposition of iron in
g to the MCQS point the liver, pancreas, joints, heart, pituitary gland, adrenals and
of view) skin.
• Slate grey skin pigmentation(skin tanning) is the early sign of
hemochrombtosis.
• Daignostic test- LFTs(liver function tests), Serum ferritin >1mg/l
is highly suggestive, Transferrin saturation >45%, Liver MRI(84-
91% sensitivity).
• Management- Venesection@ 1 unit/1-3 weeks, until ferritin
level below 50ug/l. maintenance venesection is needed to keep
Hematocrit below 0.5, serum ferritin below 100ug/l, serum
transferrin saturation below 40%. Desferroximine( iron
chelating drug) may needed in venesection intolerant patients.
• Screening test- serum ferritin or genotyping in high risk
population. broadly we can investigate patient with Iron
studies(Serum Ferritin, Iron level, Iron binding capacity)
• Prognosis is poor in cirrhotic patients as it may lead to HCC,
gonadal failure is irreversible.
• Secondary hemochromatosis in patients with heavy blood
transfusion(40litre).
Reference OHCM PG#262



47
GIT-System Wise 1700-by Sush and Team. 2016
Susmita, Asad, Manu, Saima, Zohaib, Savia, Shanu, Mona, Manisha, Sitara, Samreena, Sami and Komal


Q: 748
The body of a 65yo man who was treated for TB and bronchitis
was seen at autopsy. His legs were swollen and his liver showed
signs of a transudate fluid. What was the cause of the transudate?

a. Liver cirrhosis.

b. Alcoholic liver disease.

c. Cardiac failure.

d. Budd-chiari syndrome.

e. TB.


Clincher(s) h/o tuberculosis, bronchitis, transudate fluid.
A In this case, there are no features of cirrhosis mentioned.
Although, liver cirrhosis also causes transudative ascites.
B Same explanation as above. No h/o Alcohol in this patient.
C Old age, TB and bronchitis is suggestive of Chronic pathology,
most likely COPD. This disease causes right heart failure- Cor
pulmonale, caused by chronic pulmonary arterial hypertension.
In this particular patient, the cause is chronic lung infections and
COPD. The signs of fluid in liver(congested liver) and bilateral
ankle oedema confirms this diagnosis.

D Budd-chiari syndrome- hepatic vein obstruction caused by


thrombosis or tutor lead to congestive ischemia and hepatocyte
damage. causes are hypercoagulabe state(pills, pregnancy,
polycythemia), TB, renal, adrenal or liver tumor. Tests include
Ultrasound scan, Angioplasty(transjugular intrahepatic
portosystemic shunt). Treatment- lifelong anticoagulant drugs.
reference OHCM PG #710
E The ascitic fluid in TB is exudate and most of the times blood
stained.
KEY Option C- cardiac failure

48
GIT-System Wise 1700-by Sush and Team. 2016
Susmita, Asad, Manu, Saima, Zohaib, Savia, Shanu, Mona, Manisha, Sitara, Samreena, Sami and Komal


Additional • Transudate extravascular fluid with low protein content and low
Information specific gravity. It causes by high hydrostatic pressure and low
colloid pressure. Protein less than 45mg/dl. It is clear fluid and
does not contains large proteins like fibrogen. Diseases include
congestive cardiac failure, cirrhosis or nephrosis.
• Exudate extravascular fluid with high protein content and high
specific gravity. It causes by inflammation leads to increased
vascular permeability. Diseases include infections, malignancies,
empyema, rheumatoid pleuritis, embolism etc.
Reference cor pulmonale in OHCM PG# 194



Q:762 A 38yo man has had a liver biopsy as part of inv for abnormal LFTs.
The pathologist report states: special stains demonstrate the
presence of a very large amount of iron pigment within the
hepatocytes. What single condition is identified by the pathologists
report?


a. Alpha 1 antitrypsin deficiency


b. Hemangioma


c. Hemochromatosis


d. Hemosiderosis


e. Wilso n’s disease


Clincher(s) h/o large pigment in the hepatocyte.
A Alpha 1 antitrypsin deficiency- inherited disorder causes
emphysema in lungs and cirrhosis and hepatocellular carcinoma in
liver.
B Liver hemangioma is a benign vascular tumor of liver, slow growing
and usually asymptomatic.

49
GIT-System Wise 1700-by Sush and Team. 2016
Susmita, Asad, Manu, Saima, Zohaib, Savia, Shanu, Mona, Manisha, Sitara, Samreena, Sami and Komal


C Hemochromatosis is a hereditary disorder of iron metabolism.
Hepatocytes are loaded with iron.
D hemosiderosis is a form of iron overload resulted in the
accumulation of hemosiderin(iron storage protein). It is a
pathological picture of hemochromatosis.

E It is a disorder of hepatic copper metabolism caused by the


mutation in the gene coding for ATPase enzyme nvolved.
Presentation include Kayser-Fleischer ring on cornea, liver failure,
cirrhosis, chronic liver disease, asymetrical tremor and severe
depression.
KEY D- hemosiderosis.
Additional Refer to question 731
Information
Reference OHCM PG#262



Q:771 A 65yo man with cancer of middle 1/3 of the esophagus presents
with dysphagia. What is the most immediate management?


a. Chemotherapy


b. Radiotherapy


c. Stenting


d. Gastrostomy


e. TPN total parental neutrition


Clincher(s) Middle 1/3rd oesophageal tumor with dysphagia.
A Chemotherapy is adjuvant therapy along with surgery.

50
GIT-System Wise 1700-by Sush and Team. 2016
Susmita, Asad, Manu, Saima, Zohaib, Savia, Shanu, Mona, Manisha, Sitara, Samreena, Sami and Komal


B Radiotherapy for reducing the size of tumor.
C Stenting is a first-line option to assist swallowing.
D Gastrostomy for nutritional assitance when stenting is not
possible due to the local invasion of tumor/large size.
E TPN can be offered but not as a first line or immediate
management for dysphagia in oesophageal carcinioma.
KEY C- stenting.
Additional • either squamous cell carcinoma (SCC) or adenocarcinoma (AC).
Information • AC is seen more frequently in Caucasian populations, whereas
SCC is more frequent in people of African descent.
• Barrett’s oesophagus, which is a precursor of AC.
• The classic RED FLAG symptoms are:
Dyphagia.
Vomiting.
Anorexia and weight loss.
Symptoms of gastrointestinal-related blood loss.
• Definitive investigation- endoscopic + Biopsy.
• Investigation for staging- CT/MRI, barium swallow.
• Management- Surgery, chemotherapy and radiotherapy.
Reference Patient.info



Q:788 A 77yo woman suffered diarrhea 4d ago. What would be her
blood gas result?


a. pH =7.2, PaCO2 =8


b. pH =7.4, PaCO2 =1.5


c. pH =7.4, PaCO2 =2.6


d. pH =7.4, PaCO2 =2.8

51
GIT-System Wise 1700-by Sush and Team. 2016
Susmita, Asad, Manu, Saima, Zohaib, Savia, Shanu, Mona, Manisha, Sitara, Samreena, Sami and Komal


Clincher(s) Diarrhea 4days ago.
A Metabolic acidosis. it could be the answer if the patient has
ongoing diarrhoea.

B This is the picture of respiratory compensation of metabolic


acidosis(suitable option).
C Compensatory Metabolic acidosis
D Compensatory Metabolic acidosis.

KEY B- best suitable option.

52
GIT-System Wise 1700-by Sush and Team. 2016
Susmita, Asad, Manu, Saima, Zohaib, Savia, Shanu, Mona, Manisha, Sitara, Samreena, Sami and Komal


Additional H2 O + CO2 ↔H2 CO3 ↔H+ + HCO3-
Information
In diarrhea .. You are losing the salts and the pancreatic juice
mainly (Na,K,HCO3,PO4) .. Na & K cause electrolyte disturbance
mainly .. So PO4 & HCO3 are lost .. HCO3 is a stronger in acid base
equation , so it's the main problem .. Leaving the H cation
increasing in blood .. As you can see from the above equation ..So
losing alkali .. Developing acidosis .. That's the earliest change ..
Where ph <7.4 .. And the only problem mentioned is diarrhea so
it's metabolic acidosis .. ie no respiratory abnormality And as the
body has 2 main mechanisms for compensation ..
Respiratory(rapid) and renal(slower) so As alkalis are lost (HCO3)
.. (H) is left unopposed in the equation .. And kidney will start
reabsorbing HCO3 and PO4 To combine with H .. Takes time !! So
from the equation we don't won't CO2 to combine with H2O and
move towards producing more H !! ...... Plus CO2 is an acid itself
adding to the acidity of PH .. So now respiratory system (the
rapid) decrease CO2 through the rapid alveolar hyperventilation.
Normally, PaCO2 falls by 1-1.3 mm Hg for every 1-mEq/L fall in
serum HCO3- concentration, .. Just an info. that it also takes time
to decrease CO2 ..After 3-4 days .. The respiratory compensation
reaches it's peak with the lowest PaCO2 and not letting ph fall
below 7.4 .. But after that lungs can no longer decrease the CO2.
And the kidneys should have started reabsorbing HCO3 ..
Combing with H .. At first in favour of decrease acidosis and
increase ph. But during so Producing CO2 which starts to rise
slowly again .. Posing a bit of overload on lungs to wash it out. If
there's no intervention taken medically .. Respiratory failure
occurs due to over exhaustion of respiratory muscles .. And the
CO2 will start to accumulate in blood shifting from compensated
(by respiratory) metabolic acidosis .. To mixed metabolic acidosis
.. Both metabolic and respiratory derangement .. Ph will start to
fall below normal (failure of compensation) .

A simple method for approaching Acid-base Questions:


1 Look at the pH:
is there an acidosis or alkalosis?
• pH <7.35 is an acidosis; pH >7.45 is an alkalosis.
2 Is the CO2 abnormal? (Normal concentration 4.7–6.0kPa)
If so, is the change in keeping with the pH?
• CO2 is an acidic gas—is CO2 raised with an acidosis, lowered
with an alkalosis? If so, it is in keeping with the pH and thus
caused by a respiratory problem. If there is no change, or an
opposite one, then the change is compensatory. 3 Is the HCO3–

53
GIT-System Wise 1700-by Sush and Team. 2016
Susmita, Asad, Manu, Saima, Zohaib, Savia, Shanu, Mona, Manisha, Sitara, Samreena, Sami and Komal


abnormal? (Normal concentration 22–28mmol/L) If so, is the
change in keeping with the pH?• HCO3– is alkaline—is HCO3–
raised with an alkalosis, lowered with an acidosis? If so, the
problem is a metabolic one.

54
GIT-System Wise 1700-by Sush and Team. 2016
Susmita, Asad, Manu, Saima, Zohaib, Savia, Shanu, Mona, Manisha, Sitara, Samreena, Sami and Komal


Reference OHCM PG# 684

Q:7 7. A 60yo man presents with dysphagia and pain on swallowing both solids and liquids. A
barium
meal shows gross dilatation of the esophagus with a smooth narrowing at the lower end
of the
esophagus. What is the SINGLE most likely cause of dysphagia?
a. Achalasia
b. Myasthenia gravis
c. Esophageal carcinoma
d. Esophageal web
e. Systemic sclerosis


Clincher(s) Middle age, both solids and liquids, GROSS dilatation and narrowing at the lower end
A. Achalasia The correct answer, also, corkscrew, bird’s beak appearance on barrium
B. MG Dysphagia + weakness of the eye muscles, diplopia or fatigue by the end of the day
C. Cancer Progressive, liquid then solid, usually in older age with other clinical features of CA RAT
TAIL APPEARANCE ON BARIUM
D. Oesophageal Plummer Vinson syndrome is the formation of an oesophageal web above the aortic arc
web in association with concomitant iron deficiency in women. It is a risk factor for
oesophageal CA

Dysphagia + chronic atrophic glossitis, koilonychia, angular stomatitis and achlorhydria.


correction of anaemia

www.gpnotebook.co.uk/simplepage.cfm?ID=-1710882805
E. Systemic Dysphagia + CREST symptoms Calcinosis (subcutaneous tissues), Raynaud’s, oesophagea
Sclerosis and gut dysmotility, Sclerodactyly (swollen tight digits), and Telangiectasia.
lower oesophageal sphincter (LES) pressure is decreased. This contrasts to achalasia
where the LES pressure is increased
KEY A
Additional
Information • Achalasia typically presents in middle-age and is equally common in men and
women
Investigations
• manometry: excessive lower oesophageal sphincter tone which doesn't relax on
swallowing - considered most important diagnostic test
• barium swallow shows grossly expanded oesophagus, fluid level, 'bird's beak'
appearance. This is in contrast to the rat's tail appearance of carcinoma of the
oesophagus
• CXR: wide mediastinum, fluid level
• Gold standard - Manometry

55
GIT-System Wise 1700-by Sush and Team. 2016
Susmita, Asad, Manu, Saima, Zohaib, Savia, Shanu, Mona, Manisha, Sitara, Samreena, Sami and Komal


Treatment
• intra-sphincteric injection of botulinum toxin
• Heller cardiomyotomy for fit young patients.
• balloon dilation for old unwell patients.

Complications : Aspiration pneumonia, perforation, GERD, Oesophagus CA.

Dysphagia
The table below gives characteristic exam question features for conditions causing
dysphagia:

Dysphagia may be associated with weight loss, anorexia or vomit
during eating

Oesophageal Past history may include Barrett's oesophagus, GORD, excessiv


cancer smoking or alcohol use

Oesophagitis May be history of heartburn

Odynophagia but no weight loss and systemically well

Oesophageal There may be a history of HIV or other risk factors such as steroid
candidiasis inhaler use

Achalasia Dysphagia of both liquids and solids from the start

Heartburn

Regurgitation of food - may lead to cough, aspiration pneumonia e

Pharyngeal More common in older men


pouch
Represents a posteromedial herniation between thyropharyngeus
and cricopharyngeus muscles

Usually not seen but if large then a midline lump in the neck that
gurgles on palpation

Typical symptoms are dysphagia, regurgitation, aspiration and

56
GIT-System Wise 1700-by Sush and Team. 2016
Susmita, Asad, Manu, Saima, Zohaib, Savia, Shanu, Mona, Manisha, Sitara, Samreena, Sami and Komal


chronic cough. Halitosis may occasionally be seen

Systemic Other features of CREST syndrome may be present, namely


sclerosis Calcinosis, Raynaud's phenomenon, oEsophageal dysmotility,
Sclerodactyly, Telangiectasia


As well as oesophageal dysmotility the lower oesophageal sphincte
(LES) pressure is decreased. This contrasts to achalasia where the L
pressure is increased

Myasthenia Other symptoms may include extraocular muscle weakness or ptos


gravis
Dysphagia with liquids as well as solids

Globus May be history of anxiety


hystericus
Symptoms are often intermittent and relieved by swallowing

Usually painless - the presence of pain should warrant further


investigation for organic causes

From wikipedia:



Reference Passmedicine, Dr. Rabia, OHCM


Q: 13. A 29yo lady who is a bank manager is referred by the GP to the medical OPC

57
GIT-System Wise 1700-by Sush and Team. 2016
Susmita, Asad, Manu, Saima, Zohaib, Savia, Shanu, Mona, Manisha, Sitara, Samreena, Sami and Komal


due to a long hx of tiredness and pain in the joints. An autoimmune screen
result showed smooth muscle antibodies positive. What is the most
appropriate next inv?
a. ECG
b. TFT
c. LFT
d. Serum glucose
e. Jejunal biopsy

Clincher(s) Young female, Smooth muscle antibodies positive
A
B
C
D
E
KEY C
Additional
Information



Most definitive Diagnosis is by liver biopsy

58
GIT-System Wise 1700-by Sush and Team. 2016
Susmita, Asad, Manu, Saima, Zohaib, Savia, Shanu, Mona, Manisha, Sitara, Samreena, Sami and Komal


Reference


Q: 20. A 64yo pt has been having freq episodes of secretory diarrhea, which is
extremely watery, with large amts of mucus. A dx of villous adenoma was
made after endoscopy. What electrolyte
abnormality is most likely in this pt?
a. Hyperkalemia
b. Hypernatremia
c. Hyponatremia
d. Hypokalemia
e. Hypercalcemia


Clincher(s)
A
B
C
D
E
KEY D
Additional Electrolyte loss in diarrhoea (Na, K, Mg, Cl). But potassium loss is more marked
Information especially in severe, chronic and when associated with mucous loss. Villous
adenomas secrete protein n potassium rich mucus so hypoalbuminemia n
hypokalemia

59
GIT-System Wise 1700-by Sush and Team. 2016
Susmita, Asad, Manu, Saima, Zohaib, Savia, Shanu, Mona, Manisha, Sitara, Samreena, Sami and Komal

A metabolic (hypochloraemic) alkalosis from loss of gastric contents (pH >7.45,


decrease HCO3–) indicates severe vomiting

Diarrhoea: loss of K+ and Bicarbonate?> alkalosis
Dr. Rabia
Hypokalaemia is usually defined as a serum concentration of potassium <3.5
mmol/L. It can be classified as follows:

• Mild - 3.1 - 3.5 mmol/L


• Moderate - 2.5 - 3.0 mmol/L
• Severe - <2.5 mmol/L

Typical ECG findings when potassium is <3.0 mmol/L:

• Flat T waves
• ST depression
• Prominent U waves

NB: the QT interval may appear prolonged, but this is usually a pseudo-
prolongation as the flattened T waves merge into the U waves.

60
GIT-System Wise 1700-by Sush and Team. 2016
Susmita, Asad, Manu, Saima, Zohaib, Savia, Shanu, Mona, Manisha, Sitara, Samreena, Sami and Komal


Reference OHCM 688


Q: 63. A 32yo man presented with slow progressive dysphagia. There is past hx of
retro-sternal discomfort and he has been treated with prokinetics and H2
blockers. What is the probably dx?
a. Foreign body
b. Plummer vinson syndrome
c. Pharyngeal pouch
d. Peptic stricture
e. Esophageal Ca


Clincher(s) Young, progressive, hx of prokinetics and H2
A
B
C
D
E
KEY D
Additional Progressive pain→ PU or CA
Information The history is more compatible with PU
Reference


Q: 77. A 55yo man has had severe pain in the right hypochondrium for 24h. The

pain comes in waves and is accompanied by nausea. Nothing seems to relieve

the pain. He feels hot and sweaty but has normal temp. What is the most

appropriate next inv:

A.US Abdomen

b. ERCP

c. MRCP

d. Serum amylase

e. UGI endoscopy

61
GIT-System Wise 1700-by Sush and Team. 2016
Susmita, Asad, Manu, Saima, Zohaib, Savia, Shanu, Mona, Manisha, Sitara, Samreena, Sami and Komal


Clincher(s) Middle age, Rt hypochondrium pain, 24 hr, colicky (ie comes in waves), No
fever
A
B
C
D
E
KEY A ultra sound,
Basically, endoscopy methods are invasive
Serum amylase is specific for pancreas problems but would not tell about any
other condition. So US is much more comprehensive than amylase.
Additional Dr. Rabia
Information
Biliary colic :

The pain starts suddenly in the epigastrium or right upper quadrant (RUQ) and

may radiate round to the back in the interscapular region.

Contrary to its name, it often does not fluctuate but persists from 15 minutes

up to 24 hours, subsiding spontaneously or with analgesics.

Nausea or vomiting often accompanies the pain, which is visceral in origin and

occurs as a result of distension of the gallbladder due to an obstruction or to

the passage of a stone through the cystic duct.

Differential diagnosis include reflux, peptic ulcers, irritable bowel syndrome,

relapsing pancreatitis and tumours - eg, stomach, pancreas, colon or

gallbladder. Two or more of these conditions may overlap, so the diagnosis

may not be easy.

ULTRASOUND is the best way to demonstrate stones, being 90-95% sensitive






62
GIT-System Wise 1700-by Sush and Team. 2016
Susmita, Asad, Manu, Saima, Zohaib, Savia, Shanu, Mona, Manisha, Sitara, Samreena, Sami and Komal


Reference Browse’s



Q:692 child with increasing jaundice and pale stools. Choose the appropriate test:



Clincher(s) Jaundice, pale stools
A Endomyseal antibodies
B Tft
C LFT
D US
E
KEY US
Additional This is obstructive jaundice where US may show dilatation of common bile
Information duct or stones].
Obstructive jaundice is a condition in which there is a blockage of the flow of
bile out of the liver. This results in redirection of excess bile and its by-products
into the blood, and bile excretion from the body is incomplete. Bilirubin, a
component of bile, is yellow, which gives the characteristic yellow appearance

63
GIT-System Wise 1700-by Sush and Team. 2016
Susmita, Asad, Manu, Saima, Zohaib, Savia, Shanu, Mona, Manisha, Sitara, Samreena, Sami and Komal


of jaundice in the skin, eyes, and mucous membranes.
Causes of obstructive jaundice are

• Gallstones - most common cause


• Pancreatic cancer
• Hepatitis
• Drugs/medications
• Interstitial liver diseases
• Primary biliary cirrhosis
• Sclerosing cholangitis

Symptoms

Symptoms of obstructive jaundice include yellowing of the skin and whites of


the eyes; paler stools and darker urine; and intense itching. Other symptoms
vary, depending on the underlying cause of the obstruction although you may
also feel tired.

Diagnosis

As obstructive jaundice is a serious condition linked to high mortality rates


early diagnosis is important. If it is caught early enough a curative operation to
remove the blockage is possible, otherwise only palliative treatments that ease
the symptoms and which may prolong life are available.

Obstructive jaundice can be diagnosed through various tests including blood


tests to examine the level of bilirubin, as well as ultrasound of the liver and bile
ducts to find out the exact cause of the obstruction is. CT scans are also used
to help examine what is causing the blockage.

Treatment

Treatment depends on what is causing the obstruction; your doctor will advise
you about your treatment options. However, these might include surgical
resection; ERCP or a procedure known as Percutaneous Transhepatic
Cholangiography (PTC) with percutaneous biliary drainage. Essentially this
allows the drainage of bile that has built up in the liver.


Reference PUB MED AND NHS.CO.UK

64
GIT-System Wise 1700-by Sush and Team. 2016
Susmita, Asad, Manu, Saima, Zohaib, Savia, Shanu, Mona, Manisha, Sitara, Samreena, Sami and Komal


Q:698 A young man has diarrhea, loss of weight and flatulence for 3 days. What is the
most imp tx?


Clincher(s)
A METRONIDAZOLE
B FLUCONAZOLE
C VANCOMYCIN
D AMOXICILLIN
E
KEY METRONIDAZOLE
Additional Antibiotic induced diarhoea (OHCM: no antibiotic unless systemic). Only
Information antibiotic given in fever or immunocompromised or elderly . First line is
metronidazole. For endemic diseases or if patient not settling on antibiotics –
do culture. Here metro given due to exclusion of others. (Be careful about
metranidazol here – perhaps E is no antibiotics, which may be answer.)
Reference RABIA DOC. 390 OHCM


Q:707 57yo male presents with sudden onset severe abdominal pain and rigidity
against a 4d background of LIF pain and pyrexia. He has no PM/SHx of note
and isn’t on any meds. What is
the most likely dx?



Clincher(s)
A INTUSSUSEPTION- presentation is different , abd pain, children, diarrohea ,
ischaemic, pain comes and goes, after 12 hours pain constant, vomiting and
dehydration, US diagnostic, constipation.
B ISCHAEMIC COLON- complication, first line conservative and then if
gangrenous, then emergency
C SIGMOID VOLVULUS- sigmoid twisting around, mass, pop pop pop, no pyrexia,
no infection, mass, constipation, distension
D PERFORATED DIVERTICULUM
E PERFORATED MECKELS DIVERTICULUM (sami: like aneurysm in intestine)
KEY D
Additional Perforated diverticulum. [Sudden onset, severe abdominal pain, rigidity, left
Information iliac fossa pain and fever are in favour of perforated diverticulum].

65
GIT-System Wise 1700-by Sush and Team. 2016
Susmita, Asad, Manu, Saima, Zohaib, Savia, Shanu, Mona, Manisha, Sitara, Samreena, Sami and Komal


A Meckel's diverticulum, a true congenital diverticulum, is a slight bulge in the
small intestine present at birth and a vestigial remnant of the
omphalomesenteric duct (also called the vitelline duct or yolk stalk). It is the
most common malformation of the gastrointestinal tract and is present in
approximately 2% of the population,[1] with males more frequently
experiencing symptoms.

A memory aid is the rule of 2s:

• 2% (of the population)


• 2 feet (proximal to the ileocecal valve)
• 2 inches (in length)
• 2 types of common ectopic tissue (gastric and pancreatic)
• 2 years is the most common age at clinical presentation
• 2:1 male:female ratio
• Mostly meckels are asymptomatic
• Commonly includes
• Malaena
• Intestinal obstruction
• Volvulus
• Intussusuption
• Acute appendicitis

Diagnosis by technitium 99m per technitatescan

And CT SACN AND usg and colonoscopy if bleeding is their

66
GIT-System Wise 1700-by Sush and Team. 2016
Susmita, Asad, Manu, Saima, Zohaib, Savia, Shanu, Mona, Manisha, Sitara, Samreena, Sami and Komal


Asymptomatic during laparoscopy or laparotomy can be diagnosed

Table 1 – Complications of Meckel's Diverticulum

Complications Percentage of symptomatic Meckel’s Diverticulum (%)


Haemorrhage 20–30
Intestinal
20–25
obstruction
Diverticulitis 10–20
Umbilical anomalies ≤10
Neoplasm 0.5-2

Treatment is laparoscopic resection



Reference NHS.CO.UK,PUB MED


Q:717 In perforation of a post gastric ulcer, where will the fluid accumulate in the
peritoneal cavity?

Clincher(s)
A LEFT PARACOLIC GUTTER
B PELVIC CAVITY
C LESSER SAC
D UNDER THE DIAPHRAGM
E RIGHT PARACOLIC GUTTER
KEY C
Additional POST PERFORATIONS LEADS TO PANCREATITIS
Information

67
GIT-System Wise 1700-by Sush and Team. 2016
Susmita, Asad, Manu, Saima, Zohaib, Savia, Shanu, Mona, Manisha, Sitara, Samreena, Sami and Komal



XRAY OF PERFORATED PEPTIC ULCER SHOWING AIR UNDER DIAPHRAGM
IT MAY LEAD TO BLEEDING IF THE GASTRODUODENAL ARTERY HAS GOT
ENTRAPPED WHICH LIES BEHIND FIRST PART OF DUODENUM

AND ANTERIOR WALL PERFORATION LEADS TO PERITONITIS CALLED
Valentinos syndrome

Air under the diaphragm, due to air in stomach
Reference PUB MED


Q:728 64yo male was admitted to the medical ward with complaint of diarrhea,
abdominal pain and weight loss for few months. Exam: clubbing, perianal skin
tags and abdominal tenderness.
Colonscopy reveals transmural granulomatous inflammation involving
ileocecal junction. He was
dx with what?


Clincher(s) TYPICAL CLINICAL PRESENTATION OF CD
A CD
B UC
C BOWEL CANCER
D GASTRIC CANCER
E IBD
KEY A
Additional THE GIVEN PICTURE IS TYPICAL OF CHRONS DISEASE
Information
Crohn's disease is a long-term condition that causes inflammation of the

68
GIT-System Wise 1700-by Sush and Team. 2016
Susmita, Asad, Manu, Saima, Zohaib, Savia, Shanu, Mona, Manisha, Sitara, Samreena, Sami and Komal


lining of the digestive system.

Inflammation can affect any part of the digestive system, from the mouth to
the back passage, but most commonly occurs in the last section of the small
intestine (ileum) or the large intestine (colon).

Common symptoms can include:

• diarrhoea
• abdominal pain
• fatigue (extreme tiredness)
• unintended weight loss
• blood and mucus in your faeces (stools)
• Inflammatory bowel disease, including UC and CD, is different from
irritable bowel syndrome (IBS). Unlike IBD, IBS does not cause
inflammation, ulcers or other damage to the bowel. Instead, IBS is a
much less serious problem called a functional disorder. This means that
the digestive system looks normal but doesn't work as it should.
Symptoms of IBS may include crampy pain, bloating, gas, mucus in the
stool, diarrhea and constipation. IBS has also been called spastic colon
or spastic bowel.


Reference NHS.CO.UK

Q:1302
1302. A 14yo boy presents with recurrent abdominal pain, malaise and weight
loss over 6m. Exam: vague mass is felt in RIF. Colonoscopy shows transmural
inflammation and granulomata. What is the most appropriate management?

Clincher(s) clinchers here are transmural inflammation and granulomata )
A a. Sulfasalazine

B
C Paracetamol
D Metronidazole
E Ibuprofen
KEY D
Additional Colonoscopy indicate colonic involvement in this case of Crohn’s ( clinchers
Information here are transmural inflammation and granulomata ). Clinical experience
suggests a modest benefit of metronidazole (10 or 20 mg/kg/day) or the
combination of metronidazole and ciprofloxacin for primary or adjunctive
therapy of colonic Crohn's disease, but not for isolated small intestinal
disease.it is also to warn that Metronidazole in particular can be associated
with permanent peripheral neuropathy when used for prolonged periods,

69
GIT-System Wise 1700-by Sush and Team. 2016
Susmita, Asad, Manu, Saima, Zohaib, Savia, Shanu, Mona, Manisha, Sitara, Samreena, Sami and Komal


especially at higher doses.

Prednisolone is the drug of first choice for Crohn’s disease.


Reference Nice guidelines :
Treatment and management
Induction of remission
Monotherapy
♦ In people with a first presentation, or a single inflammatory
exacerbation of Crohn's disease in a 12-month period: First-line: conventional
glucocorticosteroids (prednisolone, methylprednisolone or IV hydrocortisone),
OR in children/young people consider enteral nutrition if there is concern
about growth or adverse effects.
♦ In people who decline, cannot tolerate or have a CI to conventional
glucocorticosteroids:
♦ Explain that budesonide and 5-ASA are less effective than conventional
glucocorticosteroids, and 5-ASA is less effective than budesonide but have
fewer adverse effects.
♦ Do NOT offer budesonide or 5-ASA for severe presentations or
exacerbations.
♦ Do NOT offer azathioprine, mercaptopurine or methotrexate as
monotherapy to induce remission.
Add-on treatment to induce remission
♦ add on treatment to be considered if

Ø there are two or more inflammatory exacerbations in a
12-month period, OR
Ø the glucocorticosteroid dose cannot be tapered.
First-line: ADD azathioprineU or mercaptopurineU to a conventional
glucocorticosteroid or budesonide:
Ø assess TPMT activity before offering azathioprine or mercaptopurine,
Ø Do NOT offer if TPMT activity is deficient (very low or absent),
Ø if TPMT activity is below normal but not deficient (according to local
laboratory reference values) consider giving azathioprine or mercaptopurine at
a lower dose.
♦ In people who cannot tolerate azathioprine or mercaptopurine, or in whom
TPMT activity is deficient.
*See Summary of Product Characteristics for full prescribing information.
Second-line: budesonide*Uchildren/young people
for people with distal ileal, ileocaecal or right-sided colonic disease, OR
5-ASA (aminosalicylates)*
mesalazineU, olsalazineU, and balsalazideU.
Second-line: ADD methotrexateU** to a conventional glucocorticosteroid or
budesonide
Tpmt activity is thiopurine methyltransferase activity done befor starting
thiopurine drugs such as azathiopurine,methylxanthine,and 6-thioguanine
because if these drugs are given in the tpmt positive drugs it may cause bone

70
GIT-System Wise 1700-by Sush and Team. 2016
Susmita, Asad, Manu, Saima, Zohaib, Savia, Shanu, Mona, Manisha, Sitara, Samreena, Sami and Komal


maroow toxicity due to accumulation of unmetabolised drugs.

CD: First line: steroid or if steroid not given and patient presents with infective
diarrohea: metro and cipro, and steroid: 5ASA.
If patient has two or more episodes over 12 month period or after trying
steroid then: add azathioprineU or mercaptopurineU to a conventional
glucocorticosteroid or budesonide:

READ 274 OHCM*** (well written)


Comparing :

Symptoms of Crohn's Disease Symptoms of Ulcerative Colitis
• Abdominal pain, cramping or swelling • Abdominal pain or discomfort
• Anemia • Anemia caused by severe bleedi
• Fever • Bloody diarrhea
• Gastrointestinal bleeding • Dehydrate
• Joint pain • Fatigue
• Malabsorption • Fever
• Persistent or recurrent diarrhea • Joint pain
• Stomach ulcers • Loss of appetite
• Vomiting • Malabsorption
• Weight loss • Rectal bleeding
• Urgent bowel movements
• Weight loss

Crohn’s disease can affect any part of the gastrointestinal tract, including the
mouth, esophagus, stomach, small and large intestines, rectum and anus.
Ulcerative colitis is located in the colon, usually starting from the rectum.
In Crohn’s disease, inflammation occurs in all layers of the intestinal wall and
patches of this inflammation can be scattered throughout the GI tract. In
contrast, in ulcerative colitis, inflammation occurs in the innermost lining of
the intestinal wall and is a continuous stretch within the colon

71
GIT-System Wise 1700-by Sush and Team. 2016
Susmita, Asad, Manu, Saima, Zohaib, Savia, Shanu, Mona, Manisha, Sitara, Samreena, Sami and Komal



Q:1305 45yo male with epigastric discomfort has been given triple therapy. He has
now returned after 4wks of epigastric discomfort. What inv would you do for
him?

Clincher(s) Age below 55 and triple therapy
A ECG
B H pylori breath test
C Endoscopy and biopsy
D US
E
KEY B
Additional As the pt is below 55 yrs old, Urea breath test should be done following a
4wks triple therapy.. If H.pylori has not been eradicated, we give triple
Information regime for one more month, and if it's normal (h pylori eradicated) then
proceed with endoscopy. Had his age been more than 55yrs, we would have
directly gone. For endoscopy
Below 55: urea breath test, above 55: endoscopy

72
GIT-System Wise 1700-by Sush and Team. 2016
Susmita, Asad, Manu, Saima, Zohaib, Savia, Shanu, Mona, Manisha, Sitara, Samreena, Sami and Komal


Reference OHCM 243

73
GIT-System Wise 1700-by Sush and Team. 2016
Susmita, Asad, Manu, Saima, Zohaib, Savia, Shanu, Mona, Manisha, Sitara, Samreena, Sami and Komal


Q:1310 A 10yo boy with lower abdominal pain for the last 10d presents with a hx of
passing 6-8 loose stools. Temp=38.8C. He is tender in the right lower quadrant

74
GIT-System Wise 1700-by Sush and Team. 2016
Susmita, Asad, Manu, Saima, Zohaib, Savia, Shanu, Mona, Manisha, Sitara, Samreena, Sami and Komal


and has an anal fistula. Choose the single most likely cause of abdominal pain.

Clincher(s) 6-8 loose stools,anal fistula
A IBD
B IBS
C Pyelonephritis
D Ureteric calculus
E Gastro enteritis
KEY IBD
Additional Anal fistula is the distinctive feature here only seen in Crohn’s (IBD)
Information
Symptoms of Crohns Disease

• Diarrhoea

• Weight loss

• Ulcers.

• Generally feeling unwell,

• Anaemia

• Mouth ulcers

• Anal fissures

• Tenesmus

Symptoms of IBS include :

• Pain and discomfort

• Bloating

• Changes in stools:

• :Some people have bouts of diarrhoea, and some have bouts of


constipation.

• Some people have bouts of diarrhoea that alternate with bouts of


constipation.

Other symptoms are:

o Feeling sick (nausea).

75
GIT-System Wise 1700-by Sush and Team. 2016
Susmita, Asad, Manu, Saima, Zohaib, Savia, Shanu, Mona, Manisha, Sitara, Samreena, Sami and Komal


o Headache.

o Belching.

o Poor appetite.

o Tiredness.

o Backache.

o Muscle pains.

o Feeling quickly full after eating.

o Heartburn.

o Bladder symptoms (an associated irritable bladder).

IBS: Manning criteria

1. Pain relieved by defecation

2. More frequent stools at onset of pain

3. Looser stools at onset of pain

4. Visible abdominal distension

5. Passage of mucus per rectum

6. Sense of incomplete evacuation

76
GIT-System Wise 1700-by Sush and Team. 2016
Susmita, Asad, Manu, Saima, Zohaib, Savia, Shanu, Mona, Manisha, Sitara, Samreena, Sami and Komal


Rome III diagnostic criteria* for irritable bowel syndrome

Recurrent abdominal pain or discomfortÀ at least 3 days a month in the past


3 months, associated with two or more of the following:

Improvement with defecation

Onset associated with a change in frequency of stool

Onset associated with a change in form (appearance) of

stool

*Criteria fulfilled for the past 3 months with symptom onset at least 6
months before diagnosis.

77
GIT-System Wise 1700-by Sush and Team. 2016
Susmita, Asad, Manu, Saima, Zohaib, Savia, Shanu, Mona, Manisha, Sitara, Samreena, Sami and Komal


À‘‘Discomfort’’ means an uncomfortable sensation not described as pain.

Alarm features in irritable bowel syndrome

Age .50 years

Short history of symptoms

Documented weight loss

Nocturnal symptoms

Male sex

Family history of colon cancer N Anaemia

Rectal bleeding

Recent antibiotic use

IBS is characterized by a combination of:

abdominal pain

cramps

constipation

diarrhea

IBD can cause the same symptoms, as well as:

eye discomfort

extreme fatigue

joint pain

rectal bleeding

Both can cause urgent bowel movements.

IBS patients may experience a feeling of incomplete evacuation as well. Pain


may be experienced across the entire abdomen. It most often manifests in

78
GIT-System Wise 1700-by Sush and Team. 2016
Susmita, Asad, Manu, Saima, Zohaib, Savia, Shanu, Mona, Manisha, Sitara, Samreena, Sami and Komal


either the lower right or lower left side. Some people will also experience
upper right side abdominal pain without any other symptoms.
IBS patients will have feeling of incomplete evacuation

Depending upon the main symptom, IBS patients are classified as


constipation-predominant, diarrhea-predominant, or pain-predominant.

PYELONEPHRITIS: present with pain ,painfull micturition,costovertebral angle


tenderness,nausea,increasedHR,and pain radiating to back


Reference Nhs.co.uk


Q:1306 1306. A 13yo boy with umbilical pain for the last 12h presents with anorexia,
nausea and has not passed a bowel motion 24h. What is your dx?


Clincher(s) 13 year boy,constipation,pain
A Acute appendicitis
B IBD
C IBS (diagnosis of exclusion> not in child)
D Meckels diverticulitis
E Muscle strain
F Ovarian cysts
G PID
H Psoas hematoma
I Pyelonephritis
J Uretric calculus

KEY A (OH 610) (Key wrong –D) This is acute appendicitis (discussed)
Additional MD occurs in <2yrs (M Diverticum/diverticulosis is congenital)
Information
its MD because it also presents with symptoms like that of Appendicitis
except rebound tenderness. Moreover, had it been AA, there would have
been hx of rebound tenderness and perhaps pain in RIF.

MD presents in 10-43% of symptomatic patients. The frequency of


complications of Meckel's diverticulum varies widely in the literature.
Studies varyingly report intestinal obstruction or haemorrhage as the most
common complication in adults.

The presenting symptoms are usually abdominal pain, vomiting and


constipation. Various mechanisms produce the obstruction, including a
fibrotic band attaching the diverticula to the abdominal wall causing a

79
GIT-System Wise 1700-by Sush and Team. 2016
Susmita, Asad, Manu, Saima, Zohaib, Savia, Shanu, Mona, Manisha, Sitara, Samreena, Sami and Komal


volvulus of the small bowel and intussusception in which the diverticulum is
the lead point. An intussusception may present with redcurrant jelly stools or
a palpable lump in the lower abdomen.Meckel's diverticulum should always
be considered in the differential diagnosis of patients presenting with rectal
bleeding or intestinal obstruction.( patient.co)

MECKELS' Presentation

Asymptomatic

Meckel's diverticulum is a common incidental finding at laparotomy. The vast


majority of those with Meckel's diverticulum are asymptomatic.
Complications are most likely to occur when the diverticulum contains
heterotypic tissue. This is most often gastric, but may also be pancreatic,
jejunal or colonic mucosa. The lifetime risk of developing a complication that
requires surgery is thought to be 4-6%.

Haemorrhage

This accounts for 25-50% of all complications. It is more common in children


younger than 2 years (in which age group it is the most common
complication) and in males. The patient usually reports bright red blood in
the stools. The amount may vary from minimal recurrent episodes to a large
shock-producing haemorrhage. Meckel's diverticulum should always be
excluded in a child presenting with massive painless rectal bleeding.

The blood may be bright red if the bleeding is brisk, or darker if it is milder
and transit time is slow. Melaena-like tarry stool may also be seen if gastric
tissue present in the diverticulum ulcerates, or if it produces acid which
causes damage to the adjacent ileal mucosa.

APPENDICITIS is most common between the ages of 10 and 20 years but can
occur at any age.

Classic symptoms often do not appear in young children, in pregnant women


and in the elderly and the diagnosis is particularly easy to miss in these age
groups. The classical presentation consists of:

Pain:

Early periumbilical pain moves, after hours or sometimes days, to the right
iliac fossa (RIF) as the peritoneum becomes involved. Pain which wakes the
patient or keeps a child awake is significant.

Movement and coughing aggravate the pain. The patient may lie still with

80
GIT-System Wise 1700-by Sush and Team. 2016
Susmita, Asad, Manu, Saima, Zohaib, Savia, Shanu, Mona, Manisha, Sitara, Samreena, Sami and Komal


shallow breathing. Deep breathing and coughing hurt.

Nausea, vomiting, anorexia. The patient is usually constipated or simply does


not want to have the bowels open, but may have diarrhoea. Rapidly
progressive cases may have recurrent vomiting without fever and diarrhoea.
This may be marked in post-ileal appendix (which is rare).


Reference Pub med,dr razia. And nhs.co.uk


Q:1314 A 40yo woman who has recently returned from working in the middle east
complains of thirst, episode of loin pain, urinary frequency, dysuria and has
passed a urinary stone. All inv are normal. She plans to return to the Middle
East in a month’s time. What is the single best advice
to prevent recurrent stone formation?

Clincher(s) Middle East,dysuria,passed urinary stone
A Drink less milk
B High fibre diet
C Increase fluid intake
D Low calcium diet
E Low protein diet
KEY C
Additional
Information The cause in this scenario seems to be dehydration as the pt lives in middle
east. She should increase her fluid intake in order to maintain adequate
urinary flow by preventing urinary stasis.
Several risk factors are recognised to increase the potential of a susceptible
individual to develop stones. These include:
• Anatomical anomalies in the kidneys and/or urinary tract - eg, horseshoe
kidney, ureteral stricture.
• Family history of renal stones.
• Hypertension.
• Gout.
• Hyperparathyroidism.
• Immobilisation.
• Relative dehydration.
• Metabolic disorders which increase excretion of solutes - eg, chronic
metabolic acidosis, hypercalciuria, hyperuricosuria.
• Deficiency of citrate in the urine.
• Cystinuria (an autosomal-recessive aminoaciduria).
• Drugs - eg, diuretics such as triamterene and calcium/vitamin D supplements.

81
GIT-System Wise 1700-by Sush and Team. 2016
Susmita, Asad, Manu, Saima, Zohaib, Savia, Shanu, Mona, Manisha, Sitara, Samreena, Sami and Komal


• More common occurrence in hot climates.
• Increased risk of stones in higher socio-economic groups.
• Contamination - as demonstrated by a spate of melamine-contaminated
infant milk formula
Reference Razia doc

Q:1540 A 69yo male presented with sudden onset of dysphagia. He is neither able to
swallow liquid nor solid, he recently had a denture fitting. What is the most
probable dx?
a. Foreign body
b. Plummer vinson syndrome
c. Achalasia cardia
d. Esophageal rupture
e. Esophageal ca


Clincher(s) Sudden onset of dysphagia,recent denture fitting
A.
B.Plummer vinson Affects mostly middle aged white women,autoimmune,genetic,Iron def
syndrome associated with post-cricoid oesophageal web or oesophageal stricture
Dysphagia is typically intermittent,begin with solids and progresses to liquids.
C.Achalasia cardia The lower oesophageal sphincter fails to relax (due to degeneration
of the myenteric plexus), causing dysphagia (for fluids and solids),
regurgitation,substernal cramps, and increase weight. CXR: fluid level in
dilated oesophagus (eg aboveheart)
D.Esophageal Clinical features Odynophagia, tachypnoea, dyspnoea, fever, shock, surgical
rupture emphysema
(a crackling sensation felt on palpating the skin over the chest or neck
caused by air tracking from the lungs.
E.Oesophageal ca Progressive dysphagia for solids and later on for liquids too,alongwith pain
and weight loss+weight loss
KEY A
Additional Esophageal foreign bodies
Information
Adults with esophageal foreign bodies usually present acutely, with a history of
ingestion. A foreign body sensation or vague discomfort in the epigastrium
suggests that the foreign body is entrapped at the LES.

Dysphagia is the norm in adults. If the obstruction is complete, an inability to


handle secretions is common. The classic adult presentation is the person with
dentures who has had some alcohol and is eating meat. Incomplete chewing
leads to an impaction at the LES. Adults should be asked about the use of
dentures, alcohol intake, and circumstances surrounding the ingestion.

http://emedicine.medscape.com/article/776566-clinical

82
GIT-System Wise 1700-by Sush and Team. 2016
Susmita, Asad, Manu, Saima, Zohaib, Savia, Shanu, Mona, Manisha, Sitara, Samreena, Sami and Komal


Reference OHCM


Q:1561 1561. A 50yo man complains of dysphagia after eating bread. Barium swallow
reveals a lower Oesophageal ring. What is the most appropriate t/m?
a. Reassurance
b. Antispasmodics
c. Dilatation of the LES (lower esophageal sphincter)
d. Endoscopic diverticulectomy
e. I&D


Clincher(s)
A Reassurance will not help the patient
B antispasmodics are given in diffuse esophageal spasm and would not be
helpful here
C
D Not relevant
E Not relevant
KEY C
Additional Esophageal rings are concentric, smooth, thin extension of normal
Information oesophageal tissue, usually 3 to 5 mm thick. They consist of mucosa,
submucosa and muscle. They may be an incidental finding at barium studies or
endoscopy. Incidence is unknown as most are asymptomatic. There is no sex
difference in the incidence of rings except that multiple rings are usually found
in young men. Rings are classified as A, B and C:
• A is uncommon and is a muscular ring several centimetres proximal to
the squamocolumnar junction. It may be an inconstant finding on
barium swallow and there is some debate as to whether it really is an
anatomical entity.
• B ring or Schatzki's ring is really a web, as it involves only mucosa and
submucosa. It tends to mark the proximal part of a hiatus hernia and
usually presents in a patient aged over 50 whose main complaint is
intermittent dysphagia to solid food, spanning months or years, and it
is non-progressive. (Most common)
• C ring is a rare X-ray finding of indentation caused by the diaphragmatic
crura. It rarely causes symptoms.


Reference Dr Rabia notes


Q:1570 A pt. presents with increasing retrosternal pain and dysphagia for both solids
and liquids over 18m but denies weight loss. Chest is clear. What is the most

83
GIT-System Wise 1700-by Sush and Team. 2016
Susmita, Asad, Manu, Saima, Zohaib, Savia, Shanu, Mona, Manisha, Sitara, Samreena, Sami and Komal


likely dx?
a. Achalasia
b. Pharyngeal carcinoma
c. Oesophageal spasm
d. Oesophageal stricture


Clincher(s) Increasing retrosternal pain, dysphagia for long time without weight loss
A Can present with retrosternal chest pain, weight loss is less in disease,
achalasia is not intermittent
B Presents with weight loss
C Presents like angina, responsive to sublingual GTN, acute problem,
D Easily diagnosed, mainly for solids.
E
KEY A or c? Manu will check with hameed
Additional · Oesophageal strictures may be benign or malignant:
Information
• Benign oesophageal strictures are usually the result of scarring from
acid reflux in severe and persistent gastro-oesophageal reflux disease
(GORD). This represents about 70 to 80% of all oesophageal strictures
and is 2 or 3 times more common in men. It may also follow ingestion
of corrosives. About 25% of patients with peptic stricture give no
history of heartburn.
• Postoperative strictures represent about 10% and corrosives account
for fewer than 5%. Drugs that can cause strictures include alendronate,
iron, non-steroidal anti-inflammatory drugs (NSAIDs), and potassium
chloride. NSAIDs should be prescribed with caution in patients with
known GORD.[3]
• Malignant oesophageal strictures usually result from carcinoma of
oesophagus but may ascend from carcinoma of stomach.


Reference Patient.info,ohcm

84
GIT-System Wise 1700-by Sush and Team. 2016
Susmita, Asad, Manu, Saima, Zohaib, Savia, Shanu, Mona, Manisha, Sitara, Samreena, Sami and Komal


Q.1659 A 60yo male is admitted with a 2d hx of lower abdominal pain and marked
vomiting. On examination he has abdominal swelling, guarding and numerous
audible bowel sounds. What is the likely dx?
a. Gallstone ileus
b. Ischemic colitis
c. Large bowel obstruction
d. Sigmoid volvulus


Clincher(s) Pain with vomiting,abdominal swelling and audible bowel sounds
A No bowel sounds in gall stone ileus
B Chronic colonic ischaemia (AKA ischaemic colitis) usually follows low flow in
the
inferior mesenteric artery (IMA) territory and ranges from mild ischaemia to
gangrenous
colitis. Presentation: Lower left-sided abdominal pain,bloody diarrhoea.
Tests: CT may be helpful but colonoscopy and biopsy is ‘gold-standard’. Barium
enema
shows characteristic ‘thumb printing’ of submucosal swelling. Treatment:
Usually
conservative with fluid replacement and antibiotics. Most recover but
strictures
are common. Gangrenous ischaemic colitis (presenting with peritonitis and
hypovolaemic
shock) requires prompt resuscitation followed by resection of the affected
bowel and stoma formation. Mortality is high.
C No bowel sounds in large bowel obstruction
D
E
KEY
Additional
Information

85
GIT-System Wise 1700-by Sush and Team. 2016
Susmita, Asad, Manu, Saima, Zohaib, Savia, Shanu, Mona, Manisha, Sitara, Samreena, Sami and Komal


Reference OHCM


Q:1699 An elderly male pt with prior hx of hematemesis is having hx of long term use
of aspirin and other drugs, now presents with severe epigastric pain, dysphagia
and vomiting. He was connected to vital monitors which were not reassuring.
What is the management?
a. Oral antacids
b. IV PPI
c. Oral PPI
d. Endoscopy
e. Analgesia
(Typical upper GI bleed features)> 55> endoscopy.

Clincher(s) History of Hematemesis,long term aspirin use,epigastric pain,dysphagia
A
B
C D
D long term use of an nsaid predisposes to peptic ulcers. the symptoms
described are of a possibly perforated peptic ulcer or acute upper Gi bleed
caused by the ulcer. The guidelines suggest that
Endoscopy is the primary diagnostic investigation in patients with acute UGIB

• Endoscopy should be undertaken immediately after resuscitation


for unstable patients with severe acute UGIB.
• Endoscopy should be undertaken within 24 hours of admission for
all other patients with UGIB.

86
GIT-System Wise 1700-by Sush and Team. 2016
Susmita, Asad, Manu, Saima, Zohaib, Savia, Shanu, Mona, Manisha, Sitara, Samreena, Sami and Komal


Treatment

Offer eradication therapy to all patients with positive tests for H. pylori.

There are several regimes. There is probably no difference between the


various PPIs available, provided that they are used at the equivalent dose and
this is a matter of personal choice.

The following is based on the recommendations of NICE.[14][18]

Recommended first-line regimes


These are optimum regimes on current evidence:

• A seven-day course of PPI plus either amoxicillin 1 g and either clarithromycin 5


metronidazole 400 mg - all three given twice a day.
• Choose the treatment regime with the lowest acquisition cost and take into acc
previous exposure to clarithromycin or metronidazole.
• For people allergic to penicillin use a PPI, clarithromycin and metronidazole - all
day for seven days.
• For people allergic to penicillin who have previously been exposed to clarithrom
use a PPI, metronidazole, tetracycline and bismuth.

Second-line H. pylori eradication regimes


• For people who do not respond to first-line therapy, offer a PPI,
amoxicillin and either clarithromycin or metronidazole (whichever was
not used first-line).

87
GIT-System Wise 1700-by Sush and Team. 2016
Susmita, Asad, Manu, Saima, Zohaib, Savia, Shanu, Mona, Manisha, Sitara, Samreena, Sami and Komal


• For people who have had previous exposure to clarithromycin and
metronidazole, offer a seven-day, twice-daily course of treatment with
a quinolone or tetracycline (whichever has the lowest acquisition cost).
• For people who are allergic to penicillin and who have not had previous
exposure to a quinolone), offer a seven-day, twice-daily course of a
PPI, and either metronidazole or levofloxacin.
• For people who are allergic to penicillin and who have had previous
exposure to a quinolone, offer a PPI, amoxicillin, bismuth,
metronidazole and tetracyclin

• For people who do not respond to first-line therapy, offer a PPI,


amoxicillin and either clarithromycin or metronidazole (whichever was
not used first-line).
• For people who have had previous exposure to clarithromycin and
metronidazole, offer a seven-day, twice-daily course of treatment with
a quinolone or tetracycline (whichever has the lowest acquisition cost).
• For people who are allergic to penicillin and who have not had previous
exposure to a quinolone), offer a seven-day, twice-daily course of a
PPI, and either metronidazole or levofloxacin.
• For people who are allergic to penicillin and who have had previous
exposure to a quinolone, offer a PPI, amoxicillin, bismuth,
metronidazole and tetracyclin

KEY D
Additional Gastric pain: pain after eating, duo: before eating pain, urea breath test,
Information guideline for triple therapy


Reference OHCM,patient.info,dr Rabia


Q: 1028 A 27yo lady after C-section developed epigastric pain after 8h. What is the
appropriate inv?
a. ABG
b. Coag profile
c. Liver enzyme
d. Liver biopsy


Clincher(s) Epigastric pain shortly after delivery.
A Not appropriate

88
GIT-System Wise 1700-by Sush and Team. 2016
Susmita, Asad, Manu, Saima, Zohaib, Savia, Shanu, Mona, Manisha, Sitara, Samreena, Sami and Komal


B Would be deranged due to liver damage in HELLP
C Liver damage due to hepato cellular necrosis… so liver enzymes appropriate
D Un necessary
E
KEY C. liver enzymes.
Additional Could be because of HELLP syndrome characterized by hemolysis, elevated
Information liver enzymes, low platelets. Usually occurs in pregnant females of pre
eclampsia and eclampsia. May present in last half of pregnancy or shortly after
delivery. Initial symptoms are non specific. Like malaise, fatigues, epigastric
pain, nausea
Investigation: CBC with peripheral smear, raised serum LDH, Bilirubin, liver
enzymes.
Rx: deliver as soon as possible
If post partum then give steroids and plasma exchange.
Complications: DIC, pulmonary edema, renal failure, liver hemorrhage and
failure. Retinal detachment

Reference


Q: 1032 A 62yo man has multiple liver mets due to adenocarcinoma with an unknown
primary. He is deeply jaundiced and has ascites with edema upto the buttocks.
He is now drowsy and his family are worried that he is not drinking enough. His
meds include: haloperidol 1.5mg, lactulose 10ml. Bloods taken 3d ago:
electrolytes normal, urea=6.5mmol/l, creatinine=89mmol/l,
calcium=2.04mmol/l, albumin=17g/L, total bilirubin=189mmol/l. What is the
single most appropriate management of his fluid intake?
a. Albumin infusion
b. Crystalloids IV
c. Crystalloids SC
d. Fluids via NGT
e. Fluids PO


Clincher(s) Liver failure with jaundice, ascites, hypoalbunemia
A.

B Restrict fluids instead
C -
D -
E -
KEY A. Albumin infusion
Additional scenario of decompensated liver disease as indicated by ascites, jaundice and
Information drowsiness. Fluid restriction should be done because of edema and ascites. His
albumin levels are low as normal value of albumin is 35-50g/L. ascites is to be

89
GIT-System Wise 1700-by Sush and Team. 2016
Susmita, Asad, Manu, Saima, Zohaib, Savia, Shanu, Mona, Manisha, Sitara, Samreena, Sami and Komal


managed with fluid restriction(less than 1.5L/day), low salt diet (40-
100mmol/d), diuretics and daily weighing.
Therapeutic paracentesis with concomitant albumin infusion (6-8g/L fluid
removed) maybe tried.


Reference


Q: 1040 A 6wk baby with vomiting, irritability and palpable mass in the abdomen on
feeding. Choose the single most likely inv?
a. Upper GI endoscopy
b. Barium meal
c. US
d. CT abdomen
e. Barium enema


Clincher(s) Age of baby, palpable mass in abdomen on feeding. – Pyloric stenosis
A
B
C Investigation of choice in pyloric stenosis
D
E
KEY B. Ultrasound
Additional scenario of infantile hypertrophic pyloric stenosis. Presents at 3-8 weeks with
Information projectile vomits esp after feed, large volume and projectile. Differentiating
point btw other causes of vomiting is that vomitus doesn’t contain bile, no
diarrhea but constipation. Alert, anxious. Hungry
o/e: left to right LUQ peristalsis during a feed. Olive sized pyloric mass present
in RUQ
labs: water and NACL deficit, hypochloremic, hypo kalemic metabolic alkalosis.
Dx. Clinical. US may be done

Rx ramsdeth’s pyloromyotomy ot endoscopic surgery

Reference


Q: 1094 A 29yo young man presents with complaints of recurrent attacks of diarrhea.
He says his stools contain blood and mucus. Sometimes he has low grade
fever. What is the most appropriate inv for his condition?
a. Stool culture
b. Plain abdominal XR
c. Per rectal exam

90
GIT-System Wise 1700-by Sush and Team. 2016
Susmita, Asad, Manu, Saima, Zohaib, Savia, Shanu, Mona, Manisha, Sitara, Samreena, Sami and Komal


d. Barium enema

Clincher(s)
A No evidence in this scenario
B Useful in acute exacerbation of UC
C
D Can not be used in acute onset of disease but investigation of choice here as
the patient not suffering from acute disease (?)
E
KEY D BARIUM ENEMA
Additional SCENARIO suggestive of Ulcerative Colitis:
Information Per rectal exam will not provide us with any evidence. Plain abdominal x ray
will be useful in an acute exacerbation of disease. Barium enema is the key but
is not an investigation of choice but can be used in acute setting to diagnose..
Here stool culture seems appropriate also so we can rule out an infectious
cause to the condition.
The diagnosis should be made on the basis of clinical suspicion supported by
appropriate macroscopic findings on sigmoidoscopy or colonoscopy, typical
histological findings on biopsy and negative stool examinations for infectious
agents.
* Initial investigations include:
* FBC, renal function and electrolytes, LFTs, ESR and CRP.
* Low magnesium and serum albumin levels are sometimes found in ulcerative
colitis. * Stool culture, including ova, cysts and parasites and also Clostridium
difficile toxin.
* Serological markers have been developed to differentiate ulcerative colitis
from Crohn's disease. p-ANCA is more commonly associated with ulcerative
colitis, whilst ASCA is more commonly associated with Crohn's disease.
* Abdominal imaging: this is essential in the initial assessment of patients with
suspected ulcerative colitis, to exclude toxic dilatation and perforation. It may
also help to assess disease extent or identify proximal constipation. In milder
forms, ultrasound, CT, MRI and radionuclide scanning may all be contributory.
* Sigmoidoscopy and rectal biopsy: for all patients presenting with diarrhoea,
rigid sigmoidoscopy should be performed unless there are immediate plans to
perform flexible sigmoidoscopy.
* Colonoscopy:
* This is usually preferable to flexible sigmoidoscopy, because the extent of
disease can be assessed but, in moderate-to-severe disease there is a higher
risk of bowel perforation and flexible sigmoidoscopy is safer

Reference


Q: 1095 A 26yo young man presents with chx of passing loose stools for the past 2m.
He says his stools contain blood and mucus and are a/w abdominal pain. He

91
GIT-System Wise 1700-by Sush and Team. 2016
Susmita, Asad, Manu, Saima, Zohaib, Savia, Shanu, Mona, Manisha, Sitara, Samreena, Sami and Komal


undergoes a colonscopy after which he was started on tx. What is the most
appropriate tx for his condition?
a. Mesalazine
b. Corticosteroids
c. Infliximab
d. Cyclosporine


Clincher(s)
A
B Can not be used for maintainence in the long term.
C In severe UC not responding to treatment
D -
E
KEY A. Mesalazine.
Additional The scenario is that of Ulcerative colitis.
Information MANAGEMENT
Aminosalicylates: Mesalazine - 5-aminosalicylic acid (5-ASA) - is now the
treatment of choice for induction and maintenance of remission of mild-to-
moderate ulcerative colitis.
Corticosteroids are used to induce remission in relapses of ulcerative colitis.
They have no role in maintenance therapy.
Thiopurines: Azathioprine and its active metabolite 6-mercaptopurine may be
used when:
* Patients are intolerant to corticosteroids.
* Patients need two or more corticosteroid courses in a calendar year.
* Disease relapses when the dose of prednisolone is less than 15 mg a day.
* Disease relapses within six weeks of stopping steroids.
Ciclosporin: This is an effective salvage therapy for patients with severe
refractory colitis and it has a rapid onset of action.
Infliximab is recommended as an option for the treatment of acute
exacerbations of severely active ulcerative colitis only in patients in whom
ciclosporin is contra-indicated or clinically inappropriate Up to 30% of patients
will ultimately require colectomy for ulcerative colitis.[3]
* Colectomy is an option for patients who do not respond to, or are intolerant
of, medical treatment, or in those with complications such as colorectal
neoplasia.[15] * Because ulcerative colitis is confined to the colorectum,
colectomy is curative.[15]
* The usual procedure is a restorative proctocolectomy with ileal pouch-anal
anastomosis.[15]

Reference UC: 2072-273 OHCM

Q:1322 A 55yr old pt presents with collapse and complains of abdominal pain that
radiates to the back. An expansile abdominal mass is felt on examination and

92
GIT-System Wise 1700-by Sush and Team. 2016
Susmita, Asad, Manu, Saima, Zohaib, Savia, Shanu, Mona, Manisha, Sitara, Samreena, Sami and Komal


pt is in shock. What is the single most likely dx?
a. Ruptured aortic aneurysm
b. Renal colic
c. Trauma
d. Endocarditis
e. atheroma

Clincher(s) Collapse, abdominal pain radiating to back, shock and expansile abd mass
A Characteristics features as described in this particular qs
B The condition classically presents with sudden-onset loin to groin pain that is
colicky in nature. Patients often complain of a sensation of not being
comfortable in any position.

It is associated with haematuria (more commonly non-visible) in more than


90% of cases

Source Gponline.com
C Trauma is one of the cause AA
D Again mycotic aneurism in endocarditis can cause AA, but symptoms of
endocarditis are murmurs, fever and past history of infection.

E This can also cause AA. Atheroma is also known as atherosclerosis and
'hardening of the arteries'. Patches of atheroma are often called plaques of
atheroma
KEY A
Additional • Ruptured AAA presents with a classical triad of pain in the flank or
Information back, hypotension and a pulsatile abdominal mass; however, only
about half have the full triad.
• The patient will complain of the pain and may feel cold, sweaty and
faint on standing.
• The following symptoms are listed with approximate frequency of
presentation:
o Abdominal pain (60%)
o Back pain (70%)
o Syncope (30%)
o Vomiting (20%)


Reference Patient info.. full detail of aortic aneurism can be found on Ohcm page 656


Q:1325 A 56 yr old woman with hx of breast cancer 10 years ago has undergone
radical mastectomy and axillary LN removal, now complains of swollen upper
limb 3 wks after an insect bite. The bite size is better but gross edema is still
present. What is the cause?

93
GIT-System Wise 1700-by Sush and Team. 2016
Susmita, Asad, Manu, Saima, Zohaib, Savia, Shanu, Mona, Manisha, Sitara, Samreena, Sami and Komal


a. Lymphedema
b. Breast Ca
c. Allergy
d. Filariasis

Clincher(s) Breast cancer hx and removal of axillary lymph node and insect bite
A
B
C
D The most spectacular symptom of lymphatic Filariasis is elephantiasis—edema
with thickening of the skin and underlying tissues—which was the first disease
discovered to be transmitted by mosquito bites.
Elephantiasis affects mainly the lower extremities. 443 OHCM
E A
KEY
Additional Lymphedema can happen days, months, or years following breast cancer
Information treatment and can be temporary or ongoing. It usually develops gradually over
time and the swelling can be mild, moderate, or severe. (Saima: Lymph cannot
drain into lymph nodes, and drains into channels and so there can be
hindrance. Patients are asked to avoid insect bite after mastectomy which can
cause severe lyphadema)
Source : breastcancer.org also Ohcm page 582
Reference


Q:1334 A woman complaining of diarrhea, abdominal pain and fatigue. All the test are
found to be normal. What is the cause
a. Somatization
b. Conversion
c. hypochondriasis

Clincher(s) Test are normal
A Clinchers are directing towards somatization as all the test are normal
B Conversion disorder is a mental condition in which a person has blindness,
paralysis, or other nervous system (neurologic) symptoms that cannot be
explained by medical evaluation.


C
Hypochondriasis also known as hypochondria, health anxiety or illness
anxiety disorder, refers to worry about having a serious illness.
D
E
KEY A
Additional Individuals with somatization disorder suffer from a number of vague physical

94
GIT-System Wise 1700-by Sush and Team. 2016
Susmita, Asad, Manu, Saima, Zohaib, Savia, Shanu, Mona, Manisha, Sitara, Samreena, Sami and Komal


Information symptoms, involving at least four different physical functions or parts of the
body. The physical symptoms that characterize somatization disorder cannot
be attributed to medical conditions or to the use of drugs, and individuals with
somatization disorder often undergo numerous medical tests (with negative
results) before the psychological cause of their distress is identified

Read more: http://www.minddisorders.com/Py-Z/Somatization-
disorder.html#ixzz3tw1c7ZHr

Reference


Q:1335 A 26yo man has returned from NY to the UK and noticed weight loss, night
sweats, temp= 37.5C and cervical lymphadenpathy. He also has
spleenomegaly. What is the dx?
a. TB
b. Lymphoma
c. Bronchial Carcinoma
d. Bronchitis

Clincher(s) Wt loss, night sweats, cervical lymphadenopathy, splenomegaly.

A No hx of cough, productive sputum or haemoptysis. (travel history)
B Travel history is for distraction but symptoms are more suggestive of
lymphoma.
C Early age and no pulmonary symptoms
D Again no features
E
KEY B

Additional In most cases of Hodgkin lymphoma, a particular cell called the Reed-Sternberg
Information cell is found when cells from the lymph node are examined during diagnosis.

This cell isn’t usually found in other types of lymphoma, so these types are
called non-Hodgkin lymphoma. (Rx are different)

This difference is important, because the treatment for Hodgkin and non-
Hodgkin lymphoma can be very different. It’s thought that Reed-Sternberg
cells are a type of white blood cell - a B-cell that has become cancerous. B-cells
normally make antibodies to fight infection.


Reference Ohcm page 354 and 356

95
GIT-System Wise 1700-by Sush and Team. 2016
Susmita, Asad, Manu, Saima, Zohaib, Savia, Shanu, Mona, Manisha, Sitara, Samreena, Sami and Komal


Q:1343 A 10yo boy is bought to the ED 10 hr after injury to the foot. It was punctured
with a metal spike that passed through his shoe. What is the next best step?
a. Ig
b. Ig and vaccine
c. Clean the wounds
d. antibiotics

Clincher(s) Injury by metal spike and punctured area
A
B
C
D
E
KEY C
Additional Initial step is always to clean the wound in order to get rid of source.
Information
Reference Dr rabia

Q: 1344 A 56yo male presents with persistent watery diarrhea. What is the most likely
dx?
a. Treponema pallidum
b. Nesseria meningitides
c. Cryptosporidium
d. Staph aureus
e. Pseudomonas aeruginosa

Clincher(s) Chronic watery
A Treponema pallidum → syphilis
B Nisseria meningitides→ meningitis
C Cryptosporidium→ a cause of persistent watery diarrhea in HIV patients
D Staph aureus→ mostly acute related to meat/food poisoning
E Pseudomonas aeruginosa→ nosocomial, immunocompromised, cystic
fibrosis. Presentation: pneumonia, septicaemia, UTI
KEY C. Cryptosporidium: Newly re-emerging infection: Fig 1. Cryptosporidium
(immunofluorescence). It is a tiny protozoan (5μm) but a big cause of
diarrhoea, esp in children in the
3rd world. It is life threatening if HIV+ve; self-limiting if CD4 ≥100;
if <100, 14L of diarrhoea can be lost/d (bad news). It’s a UK crime
to sell water if >1 oocysts/10L. Spread: unboiled water; cattle. If
found in stool, quantify excretion. If needed, ask a microbiologist (
often fails); optimizing anti-HIV may be the only
approach.83 Consider: nitazoxanide 0.5g/12h PO for 3d (if >12yrs). OHCM
390
Additional

96
GIT-System Wise 1700-by Sush and Team. 2016
Susmita, Asad, Manu, Saima, Zohaib, Savia, Shanu, Mona, Manisha, Sitara, Samreena, Sami and Komal


Information • Chronic diarrhea due to Chronic infection - eg, amoebiasis, giardiasis,
hookworm, Cryptosporidium spp., Entamoeba histolytica (may be
bloody diarrhoea). Patient.info
• Tropical malabsorption (Giardia, Cryptosporidium, Isospora belli,
Cyclospora
cayetanensis, microsporidia.) Tropical sprue: Villous atrophy +
malabsorption occurring in the Far and Middle East and Caribbean—
the cause is unknown. Tetracycline
250mg/6h PO + folic acid 15mg/d PO + optimum nutrition may
help.259 OHCM 208



• Pseudomonas aeruginosa is a serious pathogen (esp. nosocomial or if
immunocompromised, and in cystic fibrosis). It causes pneumonia,
septicaemia (risk  if: immunosuppressed, recent antibiotic use,
central venous or urinary catheter307), UTI,
wound infection, osteomyelitis, and cutaneous infections. The main
problem is its
increasing antibiotic resistance. : Piperacillin (p378) or mezlocillin +
an aminoglycoside (p766). Ciprofloxacin, ceftazidime, and imipenem
(p380) are also useful. OHCM P 422

97
GIT-System Wise 1700-by Sush and Team. 2016
Susmita, Asad, Manu, Saima, Zohaib, Savia, Shanu, Mona, Manisha, Sitara, Samreena, Sami and Komal

98
GIT-System Wise 1700-by Sush and Team. 2016
Susmita, Asad, Manu, Saima, Zohaib, Savia, Shanu, Mona, Manisha, Sitara, Samreena, Sami and Komal


Reference OHCM 247, http://patient.info/doctor/chronic-diarrhoea-in-adults


Q: 1348. A 56yo lady presents with a pathological fracture of T11 vertebra. There is
found to be an underlying metastatic lesion. What is her most common
primary ca?
a. Lung
b. Breast
c. Uterine
d. Brain

Clincher(s) Female, middle age
A
B
C
D
E
KEY B
Additional Primary tumours which give rise to metastases to bone are remembered by
Information many students by the nonsense rule that they all begin with a B. Thus:

99
GIT-System Wise 1700-by Sush and Team. 2016
Susmita, Asad, Manu, Saima, Zohaib, Savia, Shanu, Mona, Manisha, Sitara, Samreena, Sami and Komal


• breast
• bronchus
• byroid (thyroid)
• bidney (kidney)
• brostate (prostate)
• (rarely, bowel)

More than 66% of metastatic secondaries to bone arise from breast or


prostate. GPnote

Metastatic tumours

• The most common bone malignancies are metastatic carcinomas.


• They are usually multiple but may be solitary.
• The most common primaries are breast, prostate, lung, kidney and
thyroid.
• Wilms' tumour and neuroblastoma are the most common metastatic
lesions in childhood.

Patient.info

Most common tumour causing bone metastases (in descending order)

• prostate
• breast
• lung

Pssmedicine


Reference http://www.gpnotebook.co.uk/simplepage.cfm?ID=288686112
patient.info/doctor/bone-tumours


Q: 1361 . A 55yo man is having slow growing ascites. When we tap the peritoneal fluid
the protein is <25 and it is clear and yellow. What could be the origin for
ascites?
a. Budd-Chiari
b. Gastrinoma
c. Hepatoma
d. TB
e. Pancreatitis

Clincher(s) Ascietes + protein in the pritoneal fluid <25 (ie ascietes)
A Budd-Chiari Occlusion of the hepatic vein
B Gastrinoma: multiple ulcers

100
GIT-System Wise 1700-by Sush and Team. 2016
Susmita, Asad, Manu, Saima, Zohaib, Savia, Shanu, Mona, Manisha, Sitara, Samreena, Sami and Komal


C
D
E
KEY A
Additional How I remember this: Exudate (high protein in the fluid) >25 causes
Information inflammation such as infection and cancer (aggressive, erosive, bad)
Transudate <25 Benign: organs failures: Heart failure, cirrhosis, nephrotic
syndrome, hepatic venous occlusion (budd-chiari)
Dr. Rabia
Budd Chiari = occlusion of the hepatic vein by, for eg hepatoma, causes
transudative ascitis.
Such occlusion increases the hydrostatic pressure within the vessel, giving rise
to the fluid shift
from the intra vascular compartment to the interstitium with consequent
ASCITIS
If SAAG (serum-ascites albumin gradient) is more than 11g/L(exudative protein
less than 25g/L) , fluid is termed as Transudate.
Causes of high SAAG ("transudate") are:
Cirrhosis - 81% (alcoholic in 65%, viral in 10%, cryptogenic in 6%)
Heart failure - 3%
Nephrotic syndrome
Hepatic venous occlusion: Budd-Chiari syndrome or veno-occlusive disease
Constrictive pericarditis
Kwashiorkor (childhood protein-energy malnutrition)
If SAAG is less than 11g/L(exudative protein more than 25g/L) , fluid is termed
as exudate
Causes of low SAAG ("exudate") are:
Cancer (metastasis and primary peritoneal carcinomatosis) - 10%
Infection: Tuberculosis - 2% or Spontaneous bacterial peritonitis
Pancreatitis - 1%
Serositis
Hereditary angioedema
Other Rare causes:
Meigs syndrome
Vasculitis
Hypothyroidism
Renal dialysis
Peritoneum mesothelioma
Abdominal tuberculosis
Reference Dr. Rabia


Q: 1368 A man presented with a purplish swelling at the anal area. It is acutely painful
and he complains of constipation for the last 2m. What is the most appropriate
management?

101
GIT-System Wise 1700-by Sush and Team. 2016
Susmita, Asad, Manu, Saima, Zohaib, Savia, Shanu, Mona, Manisha, Sitara, Samreena, Sami and Komal


a. I&D
b. I&D + antibiotics
c. Reassure
d. Analgesia
e. Sclerotherapy
I&D: Incision and drainage
Clincher(s) Purplish swelling, acutely painful, constipation for 2m (purple because
thromobsed)
A I&D For perianal abcess, the swelling would be red and warm also, antibiotic
not needed
B I&D + AB for perianal abcess in p. w. DM or immunocompromised
C
D The answer should be excision, perhaps old or wrong recall
For the given options, analgesia is the most appropriate
E Sclerotherapy: For 2nd and 3rd degree haemorrhoid
KEY D (thrombosed haemorrhoids/will be excision if given)
Additional
Information


Management

• soften stools: increase dietary fibre and fluid intake


• topical local anaesthetics and steroids may be used to help symptoms

102
GIT-System Wise 1700-by Sush and Team. 2016
Susmita, Asad, Manu, Saima, Zohaib, Savia, Shanu, Mona, Manisha, Sitara, Samreena, Sami and Komal


• outpatient treatments: rubber band ligation is superior to injection
sclerotherapy
• surgery is reserved for large symptomatic haemorrhoids which do not
respond to outpatient treatments
• newer treatments: Doppler guided haemorrhoidal artery ligation,
stapled haemorrhoidopexy


Acutely thrombosed external haemorrhoids

• typically present with significant pain


• examination reveals a purplish, oedematous, tender subcutaneous
perianal mass
• if patient presents within 72 hours then referral should be considered
for excision. Otherwise patients can usually be managed with stool
softeners, ice packs and analgesia. Symptoms usually settle within 10
days. Passmedicine





Treatment of complications from Bailey and Love
• Strangulation, thrombosis and gangrene. In these cases,
it was formerly believed that surgery would promote portal
pyaemia. However, if adequate antibiotic cover is given from
the start, this is not found to be so, and immediate surgery
can be justified in many patients. The other risk if surgery is
performed at this stage, that of postoperative stenosis, results
in some surgeons reviewing the situation much later and

103
GIT-System Wise 1700-by Sush and Team. 2016
Susmita, Asad, Manu, Saima, Zohaib, Savia, Shanu, Mona, Manisha, Sitara, Samreena, Sami and Komal


carrying out haemorrhoidectomy only if necessary. Besides
adequate pain relief, bed rest with frequent hot baths and
warm or cold saline compresses with firm pressure usually
cause the pile mass to shrink considerably in 3–4 days (the
authors’ preference is shrinkage through external application
of small bags of frozen peas). An anal dilatation technique
has in the past been used as an alternative treatment to
surgery for painful ‘strangulated’ haemorrhoids (Figure
73.28). However, because of the risk of incontinence this is
no longer advised.

Dr. Rabia
Piles (haemorrhoids) and perianal haematoma
Anal pain can sometimes be caused by piles or a perianal haematoma (burst
blood vessel under
the skin at the edge of the anus).
Piles can become painful when they become "strangulated" and bulge outside
the anus,
developing a blood clot. They are usually treated with painkillers, ointments
and sometimes ice
packs, although surgery is occasionally needed.
A perianal haematoma is usually relieved by a simple procedure to remove the
clot using a local
anaesthetic, sometimes done by your GP and then antibiotics are given
Reference OHCM 634, webmed, passmedicine


Q: 1371 A young boy presents with fever and cough. His father was dx with TB a week
ago. The parents don’t want him to have a BAL (broncho alveolar lavage) under
anesthesia. Which other samples can be taken for dx?
a. Urine
b. Blood
c. CSF
d. Gastric washing
e. Sweat



Clincher(s)
A
B
C
D
E
KEY D

104
GIT-System Wise 1700-by Sush and Team. 2016
Susmita, Asad, Manu, Saima, Zohaib, Savia, Shanu, Mona, Manisha, Sitara, Samreena, Sami and Komal


Additional Credits to Saima ☺
Information It seems that regarding the high number of positive GL cultures (85.7%), GL can
be effective for diagnosis of patients who have suspicious tuberculosis
symptoms and are unable to produce sputum especially in resource limited
areas.
http://www.ncbi.nlm.nih.gov/pmc/articles/PMC3249987/

Goblet cells indicate metaplasia

Children swallow mucus > AFB can be found in Gastric lavage


Dr. Rabia’s explanation was wrong
“Ans D
Coz goblet cells secrete mucins n they r found in airways n stomach too”

Goblet cells are a requirement for the diagnosis of intestinal metaplasia of the
stomach. The gastric mucosa is lined by a monolayer of columnar epithelium
with some specialization at the crypts, but there are no goblet cells in normal
gastric epithelium. The appearance of goblet cells in gastric epithelium is an
indicator of potential malignant progression toward adenocarcinoma.
http://www.ncbi.nlm.nih.gov/pubmed/20059269



Imaging of goblet cells as a marker for intestinal metaplasia of the stomach
by…
ncbi.nlm.nih.gov|By Bao H , et al.

Reference

Samreena- GIT 230, 272, 283, 294, 310
Q 230.A lady comes in severe liver disease and hematemesis. Her INR is >10.
What should she be given?
a. FFP
b. Steroids
c. Whole blood
d. IV fluids
e. Vit K

FFP (fresh frozen plasma) is efficacious for treatment of deficiencies
of factors II, V, VII, IX, X, and XI when specific component therapy is neither

105
GIT-System Wise 1700-by Sush and Team. 2016
Susmita, Asad, Manu, Saima, Zohaib, Savia, Shanu, Mona, Manisha, Sitara, Samreena, Sami and Komal


available nor appropriate. Requirements for FFP vary with the specific factor
being replaced. For example, hemostatic levels of factor IX in a patient with
severe deficiency are difficult to achieve with FFP alone, whereas patients
with severe factor X deficiency require factor levels of about 10 percent to
achieve hemostasis and are easily treated with FFP.
Since in this case patient is suffering from liver disease so clotting
factors are not being synthesized by liver so FFP is appropriate. FFP
continues to be widely used for excessive anticoagulation in many areas of
the UK and other countries. This is possibly due to its low cost and the
limited availability of PCC.


Q272. A 60yo woman presented to OPD with dysphagia. No hx of weight loss
of heartburn. No change in bowel habits. While doing endoscopy there is
some difficulty passing through the LES, but no other abnormality is noted.
What is the single most useful inv?
f. CXR
g. MRI
h. Esophageal biopsy
i. Esophageal manometry
j. Abdominal XR

Esophageal achalasia is an esophageal motility disorder involving the
smooth muscle layer of the esophagus and the lower esophageal sphincter
(LES). It is characterized by incomplete LES relaxation, increased LES tone,
and lack of peristalsis of the esophagus (inability of smooth muscle to move
food down the esophagus) in the absence of other explanations like cancer or
fibrosis.
Achalasia is characterized by difficulty in swallowing, regurgitation, and
sometimes chest pain. Diagnosis is reached with esophageal manometry and
barium swallow radiographic studies.

106
GIT-System Wise 1700-by Sush and Team. 2016
Susmita, Asad, Manu, Saima, Zohaib, Savia, Shanu, Mona, Manisha, Sitara, Samreena, Sami and Komal


Because of its sensitivity, manometry (esophageal motility study) is
considered the key test for establishing the diagnosis.

Characteristic manometric findings are:

• Lower esophageal sphincter (LES) fails to relax upon wet swallow (<75%
relaxation)
• Pressure of LES <26 mm Hg is normal,>100 is considered achalasia, > 200 is
nut cracker achalasia.
• Aperistalsis in esophageal body
• Relative increase in intra-esophageal pressure as compared with intra-gastric
pressure

In case of Achalasia, either they will show a bird beak x ray or give Q
about manometry

Q283. A 47yo man who is a chronic alcoholic with established liver damage,
has been brought to the hospital after an episode of heavy drinking. His is not
able to walk straight and is complaining of double vision and is shouting
obscenities and expletives. What is the most likely dx?
k. Korsakoff psychosis
l. Delirium tremens
m. Wernickes encephalopathy
n. Tourettes syndrome e. Alcohol dependence

Wernickes Encephalopathy:
Thiamine (vitamin B-1) deficiency can result in Wernicke's
Encephalopathy (WE), a serious neurologic disorder. a triad of acute
mental confusion, ataxia, and ophthalmoplegia. Korsakoff amnestic
syndrome is a late neuropsychiatric manifestation of WE with
memory loss and confabulation; sometimes, the condition is referred
to as Wernicke-Korsakoff syndrome (WKS) or psychosis.

Although as little as 2 mg of thiamine may be enough to reverse
symptoms, the dose of thiamine required to prevent or treat WE in
most alcoholic patients may be as high as greater than 500 mg given
once or, preferably, 2 or 3 times daily parenterally. Alcohol also
appears to significantly increase the amount of thiamine required to
treat the patient successfully compared with individuals in whom
thiamine deficiency has a predominantly nutritional cause.

Wernickes (reversible) happens early and Korsakoff
(irreversible) occurs late

Q294. What is the pathological change in Barret’s esophagitis?

107
GIT-System Wise 1700-by Sush and Team. 2016
Susmita, Asad, Manu, Saima, Zohaib, Savia, Shanu, Mona, Manisha, Sitara, Samreena, Sami and Komal


o. Squamous to columnar epithelium
p. Columnar to squamous epithelium
q. Dysplasia
r. Metaplasia

Barret’s Oesophagus:
This occurs from long-standing reflux. It consists of columnar
epithelium with intestinal metaplasia extending upwards into lower
oesophagus replacing normal squamous epithelium. It is seen in up to
20% of patients undergoing endoscopy for GORD. Endoscopically,
there is finger like projection upwards from squamo-columnar
junction or island of columnar mucosa intersprea in residual
squamous mucosa.
Squamous to columnar

• frequent and longstanding heartburn


• trouble swallowing (dysphagia)
• vomiting blood (hematemesis)
• pain under the sternum where the esophagus meets the stomach
• unintentional weight loss because eating is painful

“Manisha’s”:

Barrett’s tissue has a different appearance than the normal lining of the
esophagus and is visible during endoscopy. Although this examination is very
accurate, your doctor will take biopsies from the esophagus to confirm the
diagnosis as well as look for the precancerous change of dysplasia that
cannot be seen with the endoscopic appearance alone. Taking biopsies from
the esophagus through an endoscope only slightly lengthens the procedure
time, causes no discomfort and rarely causes complications. Your doctor can
usually tell you the results of your endoscopy after the procedure, but you
will have to wait a few days for the biopsy results.


Q310.An alcoholic 56yo man had ascetic fluid analysis done which was found
to be yellow color. What is the most appropriate cause?
s. Alcoholic hepatitis
t. Decompensated cirrhosis
u. TB peritonitis
v. Pyogenic peritonitis
w. Neoplasm

Alcoholic Liver Disease


1. Fatty changes: Fatty change, or steatosis is the accumulation of fatty acids in
liver cells. These can be seen as fatty globules under the microscope. Alcoholism

108
GIT-System Wise 1700-by Sush and Team. 2016
Susmita, Asad, Manu, Saima, Zohaib, Savia, Shanu, Mona, Manisha, Sitara, Samreena, Sami and Komal


causes development of large fatty globules (macro vesicular steatosis) throughout
the liver and can begin to occur after a few days of heavy drinking.

2. Alcoholic hepatitis

Alcoholic hepatitis is characterized by the inflammation of hepatocytes.


Between 10% and 35% of heavy drinkers develop alcoholic hepatitis.

3. Cirrhosis

Cirrhosis is a late stage of serious liver disease marked by inflammation


(swelling), fibrosis (cellular hardening) and damaged membranes preventing
detoxification of chemicals in the body, ending in scarring and necrosis (cell
death). Between 10% to 20% of heavy drinkers will develop cirrhosis of the liver.
Symptoms include jaundice (yellowing), liver enlargement, and pain and
tenderness from the structural changes in damaged liver architecture. Without
total abstinence from alcohol use, will eventually lead to liver failure. Late
complications of cirrhosis or liver failure include portal hypertension (high blood
pressure in the portal vein due to the increased flow resistance through the
damaged liver), coagulation disorders (due to impaired production of coagulation
factors), ascites (heavy abdominal swelling due to buildup of fluids in the tissues)
and other complications, including hepatic encephalopathy and the hepatorenal
syndrome. Cirrhosis can also result from other causes than alcohol abuse, such as
viral hepatitis and heavy exposure to toxins other than alcohol.

In cirrhosis- Yellow ascites

109
GIT-System Wise 1700-by Sush and Team. 2016
Susmita, Asad, Manu, Saima, Zohaib, Savia, Shanu, Mona, Manisha, Sitara, Samreena, Sami and Komal




Q: 1395 A 64yo alcoholic who has been dx with liver cirrhosis presents with a massive
ascites. What is the mechanism of fluid accumulation in a pt with liver disease?
a. Cirrhosis
b. Portal HTN
c. Hypoalbuminemia
d. Liver failure
e. Hepatic encephalopathy

110
GIT-System Wise 1700-by Sush and Team. 2016
Susmita, Asad, Manu, Saima, Zohaib, Savia, Shanu, Mona, Manisha, Sitara, Samreena, Sami and Komal



Clincher(s) Mechanism of fluid accumulation
A, D & E The qn is asking for mechanism
B
C
KEY: C Dr. Rabia: This question asks about the cause of a broader term - fluid
accumulation - which is mainly mediated by hypoalbuminemia. Liver cirrhosis
is the final stage of liver disease and hypoalbuminemia can be noticed prior to
that stage.

Additional
Information


Reference OHCM




Q: 1397 A 68yo man who is a known case of liver cirrhosis has developed ascites. What
is the mechanism for the development of ascites?
a. Portal HTN
b. Hypoalbuminemia
c. Congestive heart failure
d. Liver failure


Clincher(s) Mechanism for ascites
A
B
C & D The qn is asking for mechanism for ascites
KEY: A Trigger factor for ascities is splanchnic vasodilation due to PHT.
Hypoalbuminemia plays a vital role too. According to ohcm pg 260, Portal HTN
is mentioned as a cause of ascities. Mechanism of "fluid accumulation"
(BROADER TERM ) is hypoalbuminaemia and ascites is portal HTN

111
GIT-System Wise 1700-by Sush and Team. 2016
Susmita, Asad, Manu, Saima, Zohaib, Savia, Shanu, Mona, Manisha, Sitara, Samreena, Sami and Komal


Additional
Information



Reference ohcm pg 260. Pic - http://qjmed.oxfordjournals.org/content/101/2/71

5 causes of ascites.


OHCM P 260

112
GIT-System Wise 1700-by Sush and Team. 2016
Susmita, Asad, Manu, Saima, Zohaib, Savia, Shanu, Mona, Manisha, Sitara, Samreena, Sami and Komal


Q: 1398 A man feels mild discomfort in the anal region and purulent discharge in
underpants. What is the most likely dx?
a. Feacal incontinence
b. Anal abscess
c. Fistula in ano
d. Anal tags
e. Rectal Ca
Rectal ca would have been presented with bleeding or feeling of incomplete
defecation.Some fistulla are painless specially if old.

Clincher(s) Man, mild discomfort in anal region and purulent discharge in underpants
A
B In anal abscess- there would be severe pain & fever. Abscess is confined in the
cavity and is very painful.
Perianal abscesses present as tender, inflamed, localized swellings at the anal
verge. Ischiorectal abscesses are also tender but cause a diffuse, indurated
swelling in the ischioanal fossa area. You will find your patient waiting
anxiously for you, pacing about, or on the edge of their chair: avoiding all
pressure is imperative. NB: Above the dentate line = visceral nerve innervation;
below = somatic innervation (very sensitive to pain)
C
D Anal skin tags, or rectal skin tags, are common and usually harmless growths
that hang off the skin around the outside of the anus.
E The clinical features of rectal carcinoma include: (GPnotebook)
• rectal bleeding:
o haematochezia - passage of bright red blood with bowel
movements
o the bleeding is commonly persistent but rarely copious
o may be streaked on stool or mixed with mucous
• change in bowel habit; low tumours cause slight constipation but high
tumours cause alternating constipation and diarrhoea
• tenesmus
• generalised debility and malaise
Less common features include:
• pain:
o colic, with distension and vomiting, due to lumen obstruction,
or
o pain localised to rectum, due to direct spread to surrounding
structures, especially the sacral nerves,
o rarely, pain during defaecation
• ascites
• abdominal distension
• hepatomegaly
KEY: C Dr.Rabia>> Fistula in ano is commonly seen in otherwise fit, young males.
Associations: crohn's, diabetes, obesity Causes: perianal sepsis, crohn’s. TB,

113
GIT-System Wise 1700-by Sush and Team. 2016
Susmita, Asad, Manu, Saima, Zohaib, Savia, Shanu, Mona, Manisha, Sitara, Samreena, Sami and Komal


Rectal CA. Immunocompromise
Inv : MRI, Endoanal USS

An anal fistula is an abnormal communication between two epithelial surfaces
- the perianal skin and the anal or rectal lumen.
• this abnormal tract is lined with granulation tissue
• it may harbour chronic infection resulting in continuous or intermittent
discharge through the external opening on to the skin
• soiling of underwear and skin irritation may be seen in severe cases due
to faecal material passing through the tract
In England, incidence of anal fistula is 18.4 per100,000/year. More common in
men (twice as likely to be affected) with peak around 40 years of age.
Fistulae may be complex, with several openings onto the perianal skin


Additional Gpnotebook, fistula in ano - ohcm pg: 632
Information
Reference


Q: 1403 A 45yo male alcoholic presents after a large hematemesis. He has some spider
naevi on his chest, BP=100/76mmHg, pulse=110bpm. He has a swollen
abdomen with shifting dullness.
a. Gastric ca
b. Mallory-weiss tear+
c. Esophageal ca
d. Esophageal varices
e. Esophagitis
f. Peptic ulceration

Clincher(s) 45yo, male, alcoholic, large hematemesis, spider naevi on his chest, swollen
abdomen with shifting dullness (ascites) = signs of chronic liver diseases
A Gastric ca S/s: Often non-specific. Dyspepsia (p59; for >1 month and age
≥50yrs demands investigation), weight ↓, vomiting, dysphagia, anaemia. Signs
suggesting incurable disease: epigastric mass, hepatomegaly; jaundice, ascites;
large left supraclavicular (Virchow’s) node (=Troisier’s sign); acanthosis
nigricans (p564).
B mallory weiss tear (MWS)= It is characterized by upper gastrointestinal
bleeding (UGIB) from mucosal lacerations in the upper gastrointestinal tract,
usually at the gastroesophageal junction or gastric cardia. Mallory Weiss tears
account for 4-8% of cases of UGIB. Haematemesis due to a Mallory Weiss tear
usually occurs after a prolonged or forceful bout of retching, vomiting,
coughing, straining or even hiccupping.
C Oesophageal ca S/S: Dysphagia; weight↓; retrosternal chest pain. Signs from
the upper third of the oesophagus: Hoarseness; cough (may be paroxysmal if

114
GIT-System Wise 1700-by Sush and Team. 2016
Susmita, Asad, Manu, Saima, Zohaib, Savia, Shanu, Mona, Manisha, Sitara, Samreena, Sami and Komal


aspiration pneumonia).
D
E Oesophagitis: infection or inflammation of the oesophagus.
F peptic ulceration= although the most common causes of upper GI bleeding are
peptic ulcer and oesophago-gastric varices but Helicobacter pylori infection is
associated with about 95% of duodenal ulcers and 80% of gastric ulcers. other
causes may include nsaids, pepsin, smoking,alcohol etc.symptoms commonly
include epigastric pain,nausea,dyspepsia etc
KEY: D Esophageal varices (chr liver dieases> portal hyn leads to Eoseophgal varices)
Causes of spider naevi include: (GPnotebook)
• cirrhosis - most frequently, alcoholic
• oestrogen excess - usually in association with chronic liver disease; part
of normal hepatic function is the inactivation of oestrogens
• hyperthyroidism
• rheumatoid arthritis - rarely (1)
Spider naevi may also be noted transiently in:
• viral hepatitis
• pregnancy - usually appear during the second to fifth months but
disappear in the final trimester

Hematemesis usually occur as a complication of chronic liver dz which causes
portal HTN and subsequent engorgement of the porto-systemic anastomoses.
→ Ascites (shifting dullness)



Dr. Rabia >> Answer= esophageal varices. The most common causes of upper
GI bleeding are peptic ulcer and oesophago-gastric varices. Factors that
increase the risk of variceal bleeding are the decompensation of liver disease
(ascites, esophageal varices, oedema, etc),alcohol intake, aspirin, nsaids etc
spider naevi is also one of the signs of chronic liver disease.

Acute treatment of variceal haemorrhage
• ABC
• correct clotting: FFP, vitamin K
• vasoactive agents: terlipressin is currently the only licensed vasoactive
agent and is supported by NICE guidelines. Octreotide may also be
used.
• prophylactic antibiotics reduce mortality in patients with liver cirrhosis
• endoscopy: endoscopic variceal band ligation is superior to endoscopic
sclerotherapy. NICE recommend band ligation
• Sengstaken-Blakemore tube if uncontrolled haemorrhage
• Transjugular Intrahepatic Portosystemic Shunt (TIPSS) if above
measures fail

115
GIT-System Wise 1700-by Sush and Team. 2016
Susmita, Asad, Manu, Saima, Zohaib, Savia, Shanu, Mona, Manisha, Sitara, Samreena, Sami and Komal


Prophylaxis of variceal haemorrhage
• propranolol: reduced rebleeding and mortality compared to placebo
• endoscopic variceal band ligation (EVL) is superior to endoscopic
sclerotherapy. It should be performed at two-weekly intervals until all
varices have been eradicated. Proton pump inhibitor cover is given to
prevent EVL-induced ulceration
Additional
Information


Reference OHCM & GP notebook; managing alcohol hepatitis – ohcm pg: 283


Q: 1406 A 36yo pt came with diarrhea, bleeding, weight loss and fistula. What is the
single most likely dx?
a. Colorectal ca
b. Celiac disease
c. CD
d. UC
e. IBS

Clincher(s) Towards crohns
A colorectal ca=commonly occurs in people aged 65 or more.
B celiac disease= peaks in infancy and 50-60yrs, diarrhea with weight loss or
anemia, steatorrhea, bloating, failure to thrive in children
C
D Ulcerative colitis= chronic diarrhea with or without blood and mucus, crampy

116
GIT-System Wise 1700-by Sush and Team. 2016
Susmita, Asad, Manu, Saima, Zohaib, Savia, Shanu, Mona, Manisha, Sitara, Samreena, Sami and Komal


abdominal discomfort. Extraintestinal features does not include fistula and
fissures
E IBS= no organic cause can be found in it. Only diagnose it if abdominal pain is
either relieved by defecation or associated with altered stool form and there
are more than or equal to 2 of: urgency, incomplete evacuation, and bloating
or distension, mucosal PR, worsening of symptoms after food. There is no
weight loss in it
KEY: C crohn's disease.
Diarrhea with weight loss and abdominal pain are the common symptom esp
in young patients these symptoms raise the suspicion of crohn's disease.
extraintestinal manifestations include anal fistulas, fissures and perianal
abscess etc

CROHN’S DISEASE
Crohn's disease is a form of inflammatory bowel disease. It commonly affects
the terminal ileum and colon but may be seen anywhere from the mouth to
anus.

Pathology
• cause is unknown but there is a strong genetic susceptibility
• inflammation occurs in all layers, down to the serosa. This is why
patients with Crohn's are prone to strictures, fistulas and adhesions

Crohn's disease typically presents in late adolescence or early adulthood.
Features include:
• presentation may be nonspecific symptoms such as weight loss and
lethargy
• diarrhoea: the most prominent symptom in adults. Crohn's colitis may
cause bloody diarrhea
• abdominal pain: the most prominent symptom in children
• perianal disease: e.g. Skin tags or ulcers
• extra-intestinal features are more common in patients with colitis or
perianal disease

Common to both Crohn's


disease (CD) and Ulcerative
colitis (UC) Notes

Related to Arthritis: pauciarticular, Arthritis is the most
disease asymmetric common extra-intestinal
activity Erythema nodosum feature in both CD and UC
Episcleritis Episcleritis is more common
Osteoporosis in CD

117
GIT-System Wise 1700-by Sush and Team. 2016
Susmita, Asad, Manu, Saima, Zohaib, Savia, Shanu, Mona, Manisha, Sitara, Samreena, Sami and Komal


Unrelated to Arthritis: polyarticular, Primary sclerosing
disease symmetric cholangitis is much more
activity Uveitis common in UC
Pyoderma gangrenosum Uveitis is more common in
Clubbing UC
Primary sclerosing cholangitis

Additional
Information
Reference OHCM

Q: 1443 . A 45yo woman presents with pruritis. Exam: skin pigmentation. Inv: raised
ALP and presence of anti-mitochondrial antibodies. What is the single most
likely dx?
a. Psoriasis
b. Scabies
c. Atopic eczema
d. Dermatitis herpetiformis
e. Hyperthyroidism
f. Primary biliary cirrhosis


Clincher(s)
A Rash on extensor surfaces-t-cell med inflammatory dermtosis characterized by
erythematous silvery plagues – usually in puberty /young adolescent

B Parasitic infect-caused by Sarcoptes scabiei- intene pruritis especially at night


and after hot showers , erythematous pimple like papules with linear tracks
representing burrows of mite – hands axilla and genital are common places
C Atopic eczema, also known as atopic dermatitis, is the most common form of
eczema. It mainly affects children, but can also affect adults.
Eczema is a condition that causes the skin to become itchy, red, dry and
cracked. It is a long-term (chronic) condition in most people, although it can
improve over time, especially in children.
Atopic eczema can affect any part of the body, but the most common areas to
be affected are:backs or fronts of the knees outside or inside of the elbows
around the neck and hands ,cheeks, scalp,


D Dermatitis herpetiformis (DH) is an autoimmune blistering disorder associated
with a gluten-sensitive enteropathy (GSE) Dermatitis herpetiformis is
characterized by grouped excoriations; erythematous, urticarial plaques; and
papules with vesicles. The classic location for dermatitis herpetiformis lesions
is on the extensor surfaces of the elbows, knees, buttocks, and back.

118
GIT-System Wise 1700-by Sush and Team. 2016
Susmita, Asad, Manu, Saima, Zohaib, Savia, Shanu, Mona, Manisha, Sitara, Samreena, Sami and Komal


Dermatitis herpetiformis is exquisitely pruritic, and the vesicles are often
excoriated (related coeliac disease)
E Skin pigmentation ,Xanthelasma (or xanthelasma palpebrarum) is a sharply
demarcated yellowish deposit of fat underneath the skin, usually on or around
the eyelids. , xanthomata
KEY
F- PRIMARY BILIARY CIRRHOSIS
Middle aged women presenting with pruritis, skin pigmentation and the ALP is
raised along with the presence of anti microbial antibodies all these features
point towards the diagnosis.

Additional Primary biliary cirrhosis is a chronic liver disorder typically seen in middle-aged
Information females (female:male ratio of 9:1).
The aetiology is not fully understood although it is thought to be an
autoimmune condition. Interlobular bile ducts become damaged by a chronic
inflammatory process causing progressive cholestasis which may eventually
progress to cirrhosis. The classic presentation is itching in a middle-aged
woman
Antimitochondrial antibodies (AMA) are the hallmark of PBC. (Alk
phosphatase increased)

Risk if:
+ve family history, many UTIS; smoking; past pregnancy; other autoimmune
diseases; use of nail polish/hair dye.

Features:
Lethargy, sleepiness, and pruritus may precede jaundice by years, Jaundice;
skin pigmentation; xanthelasma; xanthomata; hepatosplenomegaly

Associations:
Sjogren's syndrome (seen in up to 80% of patients)
rheumatoid arthritis
systemic sclerosis
thyroid disease
Tests:
Blood: ↑Alk phos,↑gamma GT, and mildly↑ AST & ALT; late disease:
↑bilirubin,↓ albumin, ↑prothrombin time
anti-mitochondrial antibodies (AMA) M2 subtype are present in 98% of
patients and are highly specific
smooth muscle antibodies in 30% of patients
raised serum IgM

Management:
pruritus: cholestyramine
fat-soluble vitamin supplementation

119
GIT-System Wise 1700-by Sush and Team. 2016
Susmita, Asad, Manu, Saima, Zohaib, Savia, Shanu, Mona, Manisha, Sitara, Samreena, Sami and Komal


ursodeoxycholic acid
liver transplantation e.g. if bilirubin > 100 (PBC is a major indication) -
recurrence in graft can occur but is not usually a problem

Primary sclerosing cholangitis: related to UC: diarrhea background will be
shown (ANCA – normally found in autoimmune disease, ANA, SMA). There will
be pain. Fever, infection, OHCM 267
Reference


Q: 1455 . A 76yo man presents with sore throat, local irritation by hot food, dysphagia
and a sensation of a lump in his throat. He has a 20y hx of smoking. What is
the single most likely dx?
a. Nasopharyngeal ca
b. Pharyngeal ca
c. Sinus squamous cell ca
d. Squamous cell laryngeal ca
e. Hypopharyngeal ca
.




Clincher(s) Elderly, sore throat , dysphagia, sensation of lump, smoking irritation
A Nasopharyngeal ca= most likely to cause a lump in the neck but may also cause
nasal obstruction, deafness and postnasal discharge.

B Pharyngeal carcinoma- oropharynx , nasopharynx , hypopharynx ca
C Sinus squamous cell carcinoma (SCC) is the most frequent type of malignant
tumor in the nose and paranasal sinuses (70%–80%). (early signs
unilateralepistaxis, nasal obstruction late symptoms such as severe intractable
headache, visual disturbance, or cranial neuropathy) occur, the neoplasm is
often advanced.

D • Laryngeal ca – ( occurs on glottis /vocal cords- so hoarseness will
predominate )Associated with smoking and heavy use of alcohol .
persistent sore throat, odynophagia ,dusphagia, bad breath, lumps in the
neck. persistent ear pain (called referred pain; the source of the pain is not
the ear),

E Hypopharyngeal ca= problems with swallowing and ear pain are common
symptoms and hoarseness +/-

120
GIT-System Wise 1700-by Sush and Team. 2016
Susmita, Asad, Manu, Saima, Zohaib, Savia, Shanu, Mona, Manisha, Sitara, Samreena, Sami and Komal


KEY Answer= B. Pharyngeal carcinoma. elderly, sore throat, dysphagia and
sensation of lump in the throat are the features of pharyngeal carcinoma
Additional Pharyngeal carcinoma
Information Cancer of the pharynx is less common than other head and neck cancers. It
occurs in three locations: oropharynx, nasopharynx and hypopharynx.
Oropharyngeal cancer = typically affects people aged between 40 and 70 years

common symptoms: are a persistent sore throat, a lump in the mouth or
throat, pain in the ear.

Investigation: Biopsy is the only way to establish the diagnosis. A fine-needle
aspiration (FNA) or biopsy may be an alternative for a neck mass; lesions that
are harder to reach may require endoscopy.
Treatment:
surgery (using open or minimally invasive approaches for tumour resection
and reconstruction), radiotherapy, chemotherapy, or a combination of these
methods.
Esophagial:= ca: dysphasia but nothing in throat (progressive dysphagia> liq>
solid)

Ref: 241> causes of dysphagia


Reference http://www.cancer.org/cancer/laryngealandhypopharyngealcancer/detailedgu
ide/laryngeal-and-hypopharyngeal-cancer-signs-symptoms


Q:1456 A 42yo female who is obese comes with severe upper abdominal pain and
right shoulder tip pain with a temp=37.8C. She has 5 children. What is the
most probable dx?
a. ERCP
b. LFT

121
GIT-System Wise 1700-by Sush and Team. 2016
Susmita, Asad, Manu, Saima, Zohaib, Savia, Shanu, Mona, Manisha, Sitara, Samreena, Sami and Komal


c. Serum amylase
d. MRCP
e. US abdomen
.



Clincher(s)
Female , 40’s , obese ,5 children upper abd pain ,right shoulder tip pain ,
A Pancreatitis
B Hepatitis
C Pancreatitis
D
E
KEY Answer= E. US Abdomen. Female, fat, fertile and forty with upper abdominal
pain points towards the diagnosis of gall stones for which investigation of
choice is US abdomen
Additional Gallstones:
Information Pigment stones: (<10%)
Small, friable, and irregular. Causes: haemolysis.


Cholesterol stones: Large, often solitary. Causes: age, obesity (Admirand’s
triangle: ↑risk of stone if lecithin↓, bilesalts↓,↑ cholesterol).
Mixed stones: Faceted (calcium salts, pigment, and cholesterol).
Gallstone prevalence: 8% of those over 40yrs. 90% remain asymptomatic.
Risk factors for stones becoming symptomatic: smoking; parity.

Acute cholecystitis follows stone or sludge impaction in the neck of the
gallbladder, which may cause continuous epigastric or RUQ pain (referred to
the right shoulder, vomiting, fever, local peritonism, or a GB mass. If the stone
moves to the common bile duct (CBD), obstructive jaundice and cholangitis
may occur

Tests: ↑WCC,
Ultrasound Is the best way to demonstrate stones, being 90-95% sensitive.
it shows a thick-walled, shrunken GB (also seen in chronic disease)

Treatment:
NBM, pain relief, IVI, and eg cefuroxime 1.5g/8h IV.

122
GIT-System Wise 1700-by Sush and Team. 2016
Susmita, Asad, Manu, Saima, Zohaib, Savia, Shanu, Mona, Manisha, Sitara, Samreena, Sami and Komal


Laparoscopic cholecystectomy is the treatment of choice for all patients fit
for GA and less than 72hrs or otherwise after 6-12wkks
Open surgery is required if there is GB perforation.
Cholecystostomy is also the preferred treatment for acalculous cholecystitis


Reference Pg 636 ohcm


Q:1499 A 62yo man has had anorectal pain aggravated by defecation for 3d. Rectal
exam: purple, tender lump at the anal verge. Flexible sigmoidoscopy: normal
rectal mucosa and hard feces. What is the best management strategy?
Question ????
a. Anal hematoma
b. Anal fissure
c. Rectal ca
d. Diverticulitis
e. Angiodysplasia.




Clincher(s) 62 yr , pain aggrav by defeacat, 3 day hx , exam findings
A Perianal haematoma (AKA thrombosed external pile) It appears as a 2–4mm
‘dark blueberry’ under the skin at the anl margian
B Very Painful – The salient features of fissure in ano are:

• history of pain of defaecation - often first occurs during a period of


constipation
• the passing of stools may cause sphincter spasm and acute pain during
defaecation, which may persist for up to an hour
• there is often fresh rectal bleeding at defaecation; the patient
complains of 'bright red blood on the paper'

On examination:

• there is a longitudinal tear that nearly always occurs directly posteriorly


and in the midline
• a sentinel pile may be seen if the anus is inspected with the buttocks
gently parted; the sentinel pile is the torn, bunched-up strip of mucosa
at the base of the fissure


C Painless – bleeding pr , wt loss , alt bowel habits
D Elderly , pain , tenderness,fever , increase in wbc in blood –confirmed by

123
GIT-System Wise 1700-by Sush and Team. 2016
Susmita, Asad, Manu, Saima, Zohaib, Savia, Shanu, Mona, Manisha, Sitara, Samreena, Sami and Komal


ctscan . barium nd endoscopy contra indicated ( outpocketing in colon)
E Colonic angiodysplasia is a common cause of acute or chronic rectal bleeding
and iron deficiency anaemia.

Angiodysplasias are tiny - 1-5 mm in diameter - hamartomatous capillary


lesions in the colonic wall which produce bleeding out of proportion to their
size. They are believed to be acquired, possibly as a result of tension on the
veins where they pass through the muscularis.

Diagnosis: 1)subtraction mesenteric arteriography may demonstrate


bleeding if rapid

2) colonscopy: may visualise lesion


KEY Answer= A. Anal Hematoma. anorectal pain aggravated by defecation and
tender lump at the anal verge point towards the diagnosis
Additional
Information Perianal haematoma (AKA thrombosed external pile)
Strictly, it is actually a clotted venous saccule.
It appears as a 2–4mm ‘dark blueberry’ under the skin at the anl margian. It
may be evacuated under LA or left to resolve spontaneously.



Haemorrhoidal tissue is part of the normal anatomy which contributes to anal
continence. These mucosal vascular cushions are found in the left lateral, right
posterior and right anterior portions of the anal canal (3 o'clock, 7'o'clock and
11 o'clock respectively). Haemorrhoids are said to exist when they become
enlarged, congested and symptomatic

Clinical features
• painless rectal bleeding is the most common symptom
• pruritus
• pain: usually not significant unless piles are thrombosed
• swellling may occur with third or fourth degree piles


Types of haemorrhoids

External
• originate below the dentate line
• prone to thrombosis, may be painful


Internal

124
GIT-System Wise 1700-by Sush and Team. 2016
Susmita, Asad, Manu, Saima, Zohaib, Savia, Shanu, Mona, Manisha, Sitara, Samreena, Sami and Komal


• originate above the dentate line
• do not generally cause pain

Grading of internal haemorrhoids

Grade I Do not prolapse out of the anal canal

Grade II Prolapse on defecation but reduce


spontaneously

Grade III Can be manually reduced

Grade Cannot be reduced


IV


Management – conservative rx
• soften stools: increase dietary fibre and fluid intake
• topical local anaesthetics and steroids may be used to help symptoms
• outpatient treatments: rubber band ligation is superior to injection
sclerotherapy
• surgery is reserved for large symptomatic haemorrhoids which do not
respond to outpatient treatments
• other: Doppler guided haemorrhoidal artery ligation, stapled
haemorrhoidopexy is recommended bcz of shorter hosp stay ,less side
effects

Acutely thrombosed external haemorrhoids
• typically present with significant pain
• examination reveals a purplish, oedematous, tender subcutaneous
perianal mass

• if patient presents within 72 hours then referral should be considered
for excision. Otherwise patients can usually be managed with stool
softeners, ice packs and analgesia. Symptoms usually settle within 10
days



Reference Pg 634 ohcm

125
GIT-System Wise 1700-by Sush and Team. 2016
Susmita, Asad, Manu, Saima, Zohaib, Savia, Shanu, Mona, Manisha, Sitara, Samreena, Sami and Komal


http://www.gpnotebook.co.uk/simplepage.cfm?ID=-208011239


Reference

Q:1507 1507. A 34yo man has an intermittent epigastric pain for 3wks. It is worse by
food but helped by some tablets he obtained from the pharmacy. He had a
similar episode 3yrs ago and his doctor gave him a course of 3 types of tablets
at the time. What is the most appropriate next inv?
a. Abdomen US
b. Barium meal
c. Serum H.Pylori antibodies
d. C13 urea breath test
e. Upper GI endoscopy



Clincher(s) Age , Intermittent epigastric pain , worsen by food , triple therapy – dx is NICE
dyspepsia bcz of h.pylori ADV :
If less
A Us- than 55yr
old
B Test for
C Serum h. pylori antibodies 92% sensitive and 83% specificity h.pylori
.....
D C13 urea breath testmost accurate NON INVASIVE used as test for If greater
eradication and 95% sens and 95% specif than 55yr
& new
E Endoscopy done ACCORD TO NICE guideline dyspepsi
a for
KEY Answer: D more thn
4-6
weeks or
Additional First step is to test for h.pylori (urea breath test ) to check eradication if less alarm
Information than 55yr old pt , if h.pylori eradicated then consider upper GI endoscopy else symp
refer for
if not eradicated rx again endoscop
If greater 55yr old or ALARM symp then do upper GI Endoscopy – if no alarm y

symptoms then stop drugs casuing dyspepsia, lifestyle changes , antacids,


review after 4weeks
Reference OCHM dyspepsia ALARMS protocol


Q: 793 A 25yo male with a hx of frequent binge drinking presents 4h after having had
a take away meal following a nights heavy drinking. He complains of nausea
and has vomited on several occasions. After the last vomiting episode, he
vomited approximately a cupful of blood. On admission, he smells of alcohol,
pulse=100bpm, BP=140/80mmHg. He has some tenderness in the epigastrium.
What is your dx?

a. Gastric carcinoma
b. Mallory-weiss tear

126
GIT-System Wise 1700-by Sush and Team. 2016
Susmita, Asad, Manu, Saima, Zohaib, Savia, Shanu, Mona, Manisha, Sitara, Samreena, Sami and Komal


c. Esophageal carcinoma
d. Esophageal varices
e. Esophageal varices
f. Peptic ulceration



Clincher(s) Alcoholic, repeated vomiting, followed by haematesis
A Hx of weight loss, age,
B
C
D
E Presentation is acute not chronic (some signs of chronic diseases) ,
KEY B
Additional The key is Mallory-weiss tear. [Repeated retching and vomiting is a recognized
Information cause of Mallory-weiss tear which is supported here by vomiting a cupful of
blood].

Alcoholic patient with the complaints of several episodes of vomiting followed
by hematemesis all point towards the diagnosis of mallory weiss tear.

Mallory–Weiss tear
Persistent vomiting/retching causes haematemesis via an oesophageal
mucosal tear


Aetiology:
Haematemesis due to a Mallory Weiss tear usually occurs after a prolonged or
forceful bout of retching, vomiting, coughing, straining or even hiccupping.

Risk factors:
excessive alcohol consumption. Conditions predisposing to retching and
vomiting: e.g gastroenteritis, hyperemesis gravidarum,bulimia, renal
disease,hepatitis, migraine, raised icp etc.

Presentation:
The classic presentation is of haematemesis following a bout of retching or
vomiting. However, a tear may occur after a single vomit. Other symptoms
include melaena, light-headedness, dizziness, or syncope, and features
associated with the initial cause of the vomiting - eg, abdominal pain

Diagnosis:
Endoscopy is the primary diagnostic investigation.

Treatment:

127
GIT-System Wise 1700-by Sush and Team. 2016
Susmita, Asad, Manu, Saima, Zohaib, Savia, Shanu, Mona, Manisha, Sitara, Samreena, Sami and Komal


Resuscitation is a priority - maintain airway, provide high-flow oxygen, correct
fluid losses.
MWS usually follows a benign course but occasionally endoscopic treatment
(ablation) is required to stop bleeding.

<139/89mm hg is normal (See nice Guidelines)



Reference Rabia


Q: 872 A 26yo presents with prolonged constipation, blood on side of stool and very
painful defecation. PR exam: very painful. What is the single most likely dx?
a. Ca Colon
b. UC
c. CD
d. Anal fissure
e. Constipation


Clincher(s) Prolonged constipation, blood on stool, very painful PR
A
B
C
D
E
KEY Ans. The key is D. Anal fissure. Spasm of anal sphincter. Local injury of tissue
due to anal sphincter.

Additional Treatment: Local smooth muscle relaxant (GTN) with antibiotics, otherwise
Information sphicterotmy to relax the sphincter (suturing in anal prolapse)- see book
Reference


Q: 891 An elderly woman is found anemia. As part of her exam, she had a barium
enema which reveals a mass lesion in the ascending colon. What is the single
most appropriate dx?

a. Sigmoid volvulus
b. Anal fissure
c. Sigmoid carcinoma
d. Cecal carcinoma
e. Diverticular disease


128
GIT-System Wise 1700-by Sush and Team. 2016
Susmita, Asad, Manu, Saima, Zohaib, Savia, Shanu, Mona, Manisha, Sitara, Samreena, Sami and Komal


Clincher(s) Anaemia, mass in ascending colon (Ba enema)
A
B
C
D
E
KEY Ans. The key is D. Cecal carcinoma.
Additional [Mass in ascending colon and anaemia makes cecal carcinoma the likely
Information diagnosis from the given options]. (more common colon carcinoma) Other ca,
fresh blood.
Difference between ca of right (weight loss, anemia, general signs) and left
side of colon (presents with obstruction). Diverticular disease can be any side,
but more common left side.

Reference


Q:960 Posterior gastric ulcer got perforated leading to bleeding involving the gastro-
duodenal artery. Where would fluid accumulate in the cavity?

a. Left paracolic gutter
b. Pelvic cavity
c. First part of duodenum
d. Under the diaphragm
e. Retroperitoneal


Clincher(s)
A
B
C
D
E
KEY d. Under the diaphragm

Additional
Information
Reference


Q:1021 An asymptomatic 56yo man who has never consumed alcohol came for a
routine checkup. Exam: increased skin pigmentation, spider angioma,
cardiomegaly, S3 gallop, liver firm with 8cm span, no ascites. He is in the risk of
which condition?

129
GIT-System Wise 1700-by Sush and Team. 2016
Susmita, Asad, Manu, Saima, Zohaib, Savia, Shanu, Mona, Manisha, Sitara, Samreena, Sami and Komal


a. Cerebellar degeneration
b. Werniecke’s encephalopathy
c. Renal failure
d. Hepatoma
e. Hepatic vein thrombosis




Clincher(s)
A
B
C
D
E
KEY D: chronic liver disease signs (skin pig, liver firm, spider angioma), heart
involvement > combination. Hereditary haemochromatomasis. Only in this
condition: both liver and heart involvement. 20-30% increased risk of
hepatoma. Liver, pancreas, heart, skin, Joints (arthralgia involve), pituitary,
adrenal (7 organs involved) [eye in Wilson’s disease]
Additional
Information The patient seems to be suffering from Hereditary Haemochromatosis, there is
a defect in HFE gene which affects iron metabolism by affecting hepcidin and
results in increased absorption of iron from intestine and increased deposition
in the body tissues. HH presents itself as Skin bronzing, pancreatic
insufficiency, hepatomegaly and later cirrhosis (These patients are also at a
higher risk of developing hepatocellular carcinoma) cardiomyopathy,
neurological or psychiatric symptoms.

Reference

130

Potrebbero piacerti anche